Você está na página 1de 90

GMAT

CRITICAL REASONING TEST SECTION 1 30 Minutes 20 Questions 1. Nearly one in three subscribers to Financial Forecaster is a millionaire, and over half are in top management. Shouldnt you subscribe to Financial Forecaster no ! A reader ho is neither a millionaire nor in top management ould be most li"ely to act in accordance ith the advertisements suggestion if he or she dre hich of the follo ing #uestionable conclusions invited by the advertisement! $A% Among finance&related periodicals. Financial Forecaster provides the most detailed financial information. $'% Top managers cannot do their (obs properly ithout reading Financial Forecaster. $)% The advertisement is placed here those ho ill be li"ely to read it are millionaires. $*% The subscribers mentioned ere helped to become millionaires or (oin top management by reading Financial Forecaster. $+% ,nly those ho ill in fact become millionaires, or at least top managers, ill read the advertisement. Questions 2-3 are based on the follo ing. )ontrary to the charges made by some of its opponents, the provisions of the ne deficit&reduction la for indiscriminate cuts in the federal budget are (ustified. ,pponents should remember that the Ne *eal pulled this country out of great economic troubles even though some of its programs ere later found to be unconstitutional. -. The authors method of attac"ing the charges of certain opponents of the ne deficit&reduction la is to $A% attac" the character of the opponents rather than
1

their claim $'% imply an analogy bet een the la and some Ne *eal programs $)% point out that the opponents claims imply a dilemma $*% sho that the opponents reasoning leads to an absurd conclusion $+% sho that the Ne *eal also called for indiscriminate cuts in the federal budget

GMAT

.. The opponents could effectively defend their position against the authors strategy by pointing out that $A% the e/pertise of those opposing the la is outstanding $'% the lac" of (ustification for the ne la does not imply that those ho dre it up ere either inept or immoral $)% the practical application of the ne la ill not entail indiscriminate budget cuts $*% economic troubles present at the time of the Ne *eal ere e#ual in severity to those that have led to the present la $+% the fact that certain fla ed programs or la s have improved the economy does not prove that every such program can do so 0. 1n Millington, a city of 23,333 people, Mercedes 4edrosa, a realtor, calculated that a family ith Millingtons median family income, 5-6,333 a year, could afford to buy Millingtons median&priced 577,333 house. This calculation as based on an 11.percent mortgage interest rate and on the realtors assumption that a family could only afford to pay up to -2 percent of its income for housing. 8hich of the follo ing corrections of a figure appearing in the passage above, if it ere the only correction that needed to be made, ould yield a ne calculation sho ing that even incomes belo the median family income ould enable families in Millington to afford Millingtons median&priced house! $A% Millingtons total population as 02,333 people. $'% Millingtons median annual family income as 5-7,333 $)% Millingtons median&priced house cost 563,333 $*% The rate at hich people in Millington had to pay mortgage interest as only 13 percent. $+% 9amilies in Millington could only afford to pay up
-

to -- percent of their annual income for housing.

GMAT

2. 4sychological research indicates that college hoc"ey and football players are more #uic"ly moved to hostility and aggression than are college athletes in noncontact sports such as s imming. 'ut the researchers conclusionthat contact sports encourage and teach participants to be hostile and aggressiveis untenable. The football and hoc"ey players ere probably more hostile and aggressive to start ith than the s immers. 8hich of the follo ing, if true, ould most strengthen the conclusion dra n by the psychological researchers! $A% The football and hoc"ey players became more hostile and aggressive during the season and remained so during the off&season, hereas there as no increase in aggressiveness among the s immers. $'% The football and hoc"ey players, but not the s immers, ere a are at the start of the e/periment that they ere being tested for aggressiveness. $)% The same psychological research indicated that the football and hoc"ey players had a great respect for cooperation and team play, hereas the s immers ere most concerned ith e/celling as individual competitors. $*% The research studies ere designed to include no college athletes ho participated in both contact and noncontact sports. $+% Throughout the :nited States, more incidents of fan violence occur at baseball games than occur at hoc"ey or football games. ;.<oss= The profitability of )ompany >, restored to private o nership five years ago, is clear evidence that businesses ill al ays fare better under private than under public o nership.
.

?ulia= 8rong. A close loo" at the records sho s that > has been profitable since the appointment of a first&class manager, hich happened hile > as still in the pubic sector. 8hich of the follo ing best describes the ea" point in <osss claim on hich ?ulias response focuses! $A% The evidence <oss cites comes from only a single observed case, that of )ompany >. $'% The profitability of )ompany > might be only temporary. $)% <osss statement leaves open the possibility that the cause he cites came after the effect he attributes to it. $*% No mention is made of companies that are partly government o ned and partly privately o ned. $+% No e/act figures are given for the current profits of )ompany >.

GMAT

7. Stronger patent la s are needed to protect inventions from being pirated. 8ith that protection, manufacturers ould be encouraged to invest in the development of ne products and technologies. Such investment fre#uently results in an increase in a manufacturers productivity. 8hich of the follo ing conclusions can most properly be dra n from the information above! $A% Stronger patent la s tend to benefit financial institutions as ell as manufacturers. $'% 1ncreased productivity in manufacturing is li"ely to be accompanied by the creation of more manufacturing (obs. $)% Manufacturers ill decrease investment in the development of ne products and technologies unless there are stronger patent la s. $*% The ea"ness of current patent la s has been a cause of economic recession. $+% Stronger patent la s ould stimulate improvements in productivity for many manufacturers. 6. 8hich of the follo ing best completes the passage belo ! At large amusement par"s, live sho s are used very deliberately to influence cro d movements. @unchtime performances relieve the pressure on a par"s restaurants. +vening performances have a rather different purpose= to encourage visitors to stay for supper. 'ehind this surface divergence in immediate purpose there is the unified underlying goal of A A A A A. $A% "eeping the lines at the various rides short by dra ing off part of the cro d $'% enhancing revenue by attracting people ho come only for the live sho s and then leave the par" $)% avoiding as far as possible traffic (ams caused by
0

visitors entering or leaving the par" $*% encouraging as many people as possible to come to the par" in order to eat at the restaurants $+% utiliBing the restaurants at optimal levels for as much of the day as possible

GMAT

C.?ames eighs more than Delly. @uis eighs more than Mar". Mar" eighs less than Ned. Delly and Ned are e/actly the same eight. 1f the information above is true, hich of the follo ing must also be true! $A% @uis eighs more than Ned. $'% @uis eighs more than ?ames. $)% Delly eighs less than @uis. $*% ?ames eighs more than Mar" $+% Delly eighs less than Mar". Questions 10-11 are based on the follo ing. 4artly because of bad eather, but also partly because some ma(or pepper gro ers have s itched to high&priced cocoa, orld production of pepper has been running ell belo orld ide sales for three years. 4epper is conse#uently in relatively short supply. The price of pepper has soared in response= it no e#uals that of cocoa. 13. 8hich of the follo ing can be inferred from the passage! $A% 4epper is a profitable crop only if it is gro n on a large scale. $'% 8orld consumption of pepper has been unusually high for three years. $)% 8orld production of pepper ill return to previous levels once normal eather returns. $*% Surplus stoc"s of pepper have been reduced in the past three years. $+% The profits that the gro ers of pepper have made in the past three years have been unprecedented. 11. Some observers have concluded that the rise in the price of pepper means that the s itch by some gro ers from pepper to cocoa left those gro ers no
2

better off than if none of them had s itchedE this conclusion, ho ever, is un arranted because it can be inferred to be li"ely that $A% those gro ers could not have foreseen ho high the price of pepper ould go $'% the initial cost involved in s itching from pepper to cocoa is substantial $)% supplies of pepper ould not be as lo as they are if those gro ers had not s itched crops $*% cocoa crops are as susceptible to being reduced by bad eather as are pepper crops $+% as more gro ers turn to gro ing cocoa, cocoa supplies ill increase and the price of cocoa ill fall precipitously.

GMAT

1-. :sing computer techni#ues, researchers analyBe layers of paint that lie buried beneath the surface layers of old paintings. They claim, for e/ample, that additional mountainous scenery once appeared in @eonardo da Fincis Mona Lisa, hich as later painted over. S"eptics reply to these claims, ho ever, that >&ray e/aminations of the Mona Lisa do not sho hidden mountains. 8hich of the follo ing, if true, ould tend most to ea"en the force of the s"eptics ob(ections! $A% There is no ritten or anecdotal record that @eonardo da Finci ever painted over ma(or areas of his Mona Lisa. $'% 4ainters of da Fincis time commonly created images of mountainous scenery in the bac"grounds of portraits li"e the Mona Lisa. $)% No one "no s for certain hat parts of the Mona Lisa may have been painted by da Fincis assistants rather than by da Finci himself. $*% 1nfrared photography of the Mona Lisa has revealed no trace of hidden mountainous scenery. $+% Analysis relying on >&rays only has the capacity to detect lead&based hite pigments in layers of paint beneath a paintings surface layers. 1.. 8hile Governor Ferdant has been in office, the states budget has increased by an average of ; percent each year. 8hile the previous governor as in office, the states budget increased by an average of 11.2 percent each year. ,bviously, the austere budgets during Governor Ferdants term have caused the slo do n in the gro th in state spending. 8hich of the follo ing, if true, ould most seriously ea"en the conclusion dra n above! $A% The rate of inflation in the state averaged 13 percent each year during the previous governors term in office and . percent each year during
;

Ferdants term. $'% 'oth federal and state income ta/ rates have been lo ered considerably during Ferdants term in office. $)% 1n each year of Ferdants term in office, the states budget has sho n some increase in spending over the previous year. $*% *uring Ferdants term in office, the state has either discontinued or begun to charge private citiBens for numerous services that the state offered free to citiBens during the previous governors term. $+% *uring the previous governors term in office, the state introduced several so&called GausterityH budgets intended to reduce the gro th in state spending.

GMAT

10. 9ederal agricultural programs aimed at benefiting one group hose livelihood depends on farming often end up harming another such group. 8hich of the follo ing statements provides support for the claim above! . An effort to help feed&grain producers resulted in higher prices for their crops, but the higher prices decreased the profits of livestoc" producers. . 1n order to reduce crop surpluses and increase prices, gro ers of certain crops ere paid to leave a portion of their land idle, but the reduction as not achieved because improvements in efficiency resulted in higher production on the land in use. .Many farm or"ers ere put out of or" hen a program meant to raise the price of grain provided grain gro ers ith an incentive to reduce production by giving them surplus grain from government reserves. $A% , but not and not $'% , but not and not $)% and , but not $*% and , but not $+% ,and 12. Technological education is orsening. 4eople bet een eighteen and t enty&four, ho are (ust emerging from their formal education, are more li"ely to be technologically illiterate than some hat older adults. And yet, issues for public referenda ill increasingly involve aspects of technology. 8hich of the follo ing conclusions can be properly dra n from the statements above! $A% 1f all young people are to ma"e informed decisions on public referenda, many of them must learn more about technology. $'% Thorough studies of technological issues and
7

innovations should be made a re#uired part of the public and private school curriculum. $)% 1t should be suggested that prospective voters attend applied science courses in order to ac#uire a minimal competency in technical matters. $*%1f young people are not to be overly influenced by famous technocrats, they must increase their "no ledge of pure science. $+% ,n public referenda issues, young people tend to confuse real or probable technologies ith impossible ideals.

GMAT

1;. 1n a political system ith only t o ma(or parties, the 17. )ompanies considering ne entrance of a third&party candidate into an election cost&cutting manufacturing race damages the chances of only one of the t o processes often compare the ma(or candidates. The third&party candidate al ays pro(ected results of ma"ing attracts some of the voters ho might other ise have the investment against the voted for one of the t o ma(or candidates, but not alternative of not ma"ing voters ho support the other candidate. Since a third& the investment ith costs, party candidacy affects the t o ma(or candidates selling prices, and share of une#ually, for reasons neither of them has any mar"et remaining constant. control over, the practice is unfair and should not be 8hich of the follo ing, allo ed. assuming that each is a 1f the factual information in the passage above is realistic possibility, true, hich of the follo ing can be most reliably constitutes the most serious inferred from it! disadvantage for companies of using the method above $A% 1f the political platform of the third party is a for evaluating the financial compromise position bet een that of the t o benefit of ne ma(or parties, the third party ill dra its voters manufacturing processes! e#ually from the t o ma(or parties. $'% 1f, before the emergence of a third party, voters $A% The costs of materials ere divided e#ually bet een the t o ma(or re#uired by the ne parties, neither of the ma(or parties is li"ely to process might not be capture much more than one&half of the vote. "no n ith certainty. $)% A third&party candidate ill not capture the votes $'% 1n several years interest of ne voters ho have never voted for candidates rates might go do n, of either of the t o ma(or parties. reducing the interest costs $*% The political stance of a third party ill be more of borro ing money to radical than that of either of the t o ma(or parties. pay for the investment. $+% The founders of a third party are li"ely to be a $)% Some cost&cutting coalition consisting of former leaders of the t o processes might re#uire ma(or parties. such e/pensive investments that there ould be no net gain for many years, until the investment as paid for by savings in the manufacturing process. $*% )ompetitors that do
6

GMAT

Questions 19-20 are based on the follo ing Archaeologists see"ing the location of a legendary siege and destruction of a city are e/cavating in several possible places, including a middle and a 16. There are far fe er children available for adoption lo er layer of a large mound. than there are people ho ant to adopt. T o million The bottom of the middle layer couples are currently aiting to adopt, but in 1C6-, contains some pieces of pottery the last year for hich figures e/ist, there ere only of type ., "no n to be from a some 23,333 adoptions. later period than the time of the destruction of the city, but the 8hich of the follo ing statements, if true, most strengthens the authors claim that there are far fe er lo er layer does not. children available for adoption than there are people 1C. 8hich of the follo ing ho ant to adopt! hypotheses is best supported $A% The number of couples aiting to adopt has by the evidence above! increased significantly in the last decade. $A% The lo er layer $'% The number of adoptions in the current year is contains the remains of greater than the number of adoptions in any the city here the siege preceding year. too" place. $)% The number of adoptions in a year is $'% The legend confuses appro/imately e#ual to the number of children stories from t o different available for adoption in that period. historical periods. $*% 4eople ho see" to adopt children often go $)% The middle layer does through a long process of intervie s and not represent the period investigation by adoption agencies. of the siege. $+% 4eople ho see" to adopt children generally $*% The siege lasted for a ma"e very good parents. long time before the city as destroyed. $+% The pottery of type . as imported to the city by traders. -3. The force of the evidence cited above is most seriously ea"ened if hich of the follo ing is true!
C

invest in a ne process might reduce their selling prices and thus ta"e mar"et share a ay from companies that do not. $+% The period of year chosen for averaging out the cost of the investment might be some hat longer or shorter, thus affecting the result.

GMAT

$A% Gerbils, small animals long native to the area, dig large burro s into hich ob(ects can fall hen the burro s collapse. $'% 4ottery of types 1 and -, found in the lo er level, as used in the cities from hich, according to the legend, the besieging forces came. $)% Several pieces of stone from a lo er&layer all have been found incorporated into the remains of a building in the middle layer. $*% 'oth the middle and the lo er layer sho evidence of large&scale destruction of habitations by fire. $+% 'ronBe a/heads of a type used at the time of the siege ere found in the lo er level of e/cavation.

13

GMAT

CRITICAL REASONING TEST SECTION 2 30 Minutes 20 Questions 1. After the national speed limit of 22 miles per hour as imposed in 1C70, the number of deaths per mile driven on a high ay fell abruptly as a result. Since then, ho ever, the average speed of vehicles on high ays has risen, but the number of deaths per mile driven on a high ay has continued to fall. 8hich of the follo ing conclusions can be properly dra n from the statements above! $A% The speed limit alone is probably not responsible for the continued reduction in high ay deaths in the years after 1C70. $'% 4eople have been driving less since 1C70. $)% *river&education courses have been more effective since 1C70 in teaching drivers to drive safely. $*% 1n recent years high ay patrols have been less effective in catching drivers ho speed. $+% The change in the speed limit cannot be responsible for the abrupt decline in high ay deaths in 1C70. -. Neighboring landholders= Air pollution from the giant aluminum refinery that has been built ne/t to our land is "illing our plants. )ompany spo"esperson= The refinery is not to blame, since our study sho s that the damage is due to insects and fungi. 8hich of the follo ing, if true, most seriously ea"ens the conclusion dra n by the company spo"esperson! $A% The study did not measure the #uantity of pollutants emitted into the surrounding air by the aluminum refinery. $'% The neighboring landholders have made no change in the ay they ta"e care of their plants. $)% Air pollution from the refinery has changed the chemical balance in the plants environment,
11

allo ing the harmful insects and fungi to thrive. $*% 4ollutants that are invisible and odorless are emitted into the surrounding air by the refinery. $+% The various species of insects and fungi mentioned in the study have been occasionally found in the locality during the past hundred years.

GMAT

.. Sales ta/es tend to be regressive, affecting poor people more severely than ealthy people. 8hen all purchases of consumer goods are ta/ed at a fi/ed percentage of the purchase price, poor people pay a larger proportion of their income in sales ta/es than ealthy people do. 1t can be correctly inferred on the basis of the statements above that hich of the follo ing is true! $A% 4oor people constitute a larger proportion of the ta/paying population than ealthy people do. $'% 4oor people spend a larger proportion of their income on purchases of consumer goods than ealthy people do. $)% 8ealthy people pay, on average, a larger amount of sales ta/es than poor people do. $*% The total amount spent by all poor people on purchases of consumer goods e/ceeds the total amount spent by all ealthy people on consumer goods. $+% The average purchase price of consumer goods bought by ealthy people is higher than that of consumer goods bought by poor people. 0. <evie ing historical data, medical researchers in )alifornia found that counties ith the largest number of television sets per capita have had the lo est incidence of a serious brain disease, mos#uito&borne encephalitis. The researchers have concluded that people in these counties stay indoors more and thus avoid e/posure to the disease. The researchers conclusion ould be most strengthened if hich of the follo ing ere true! $A% 4rograms designed to control the siBe of disease& bearing mos#uito populations have not affected the incidence of mos#uito& borne encephalitis. $'% The occupations of county residents affect their ris" of e/posure to mos#uito&borne encephalitis more
1-

than does television& atching. $)% The incidence of mos#uito&borne encephalitis in counties ith the largest number of television sets per capita is li"ely to decrease even further. $*% The more time people in a county spend outdoors, the greater their a areness of the dangers of mos#uito&borne encephalitis. $+% The more television sets there are per capita in a county, the more time the average county resident spends atching television.

GMAT

2. The citys public transportation system should be removed from the (urisdiction of the municipal government, hich finds it politically impossible either to raise fares or to institute cost&saving reductions in service. 1f public transportation ere handled by a private firm, profits ould be vigorously pursued, thereby eliminating the necessity for covering operating costs ith government funds. The statements above best support the conclusion that $A% the private firms that ould handle public transportation ould have e/perience in the transportation industry $'% political considerations ould not prevent private firms from ensuring that revenues cover operating costs $)% private firms ould receive government funding if it ere needed to cover operating costs $*% the public ould approve the cost&cutting actions ta"en by the private firm $+% the municipal government ould not be resigned to accumulating merely enough income to cover costs ;. To entice customers a ay from competitors, <ed @abel supermar"ets have begun offering discounts on home appliances to customers ho spend 523 or more on any shopping trip to <ed @abel. <ed @abel e/ecutives claim that the discount program has been a huge success, since cash register receipts of 523 or more are up thirty percent since the beginning of the program. 8hich of the follo ing, if true, most seriously ea"ens the claim of the <ed @abel e/ecutives! $A% Most people ho s itched to <ed @abel after the program began spend more than 523 each time they shop at <ed @abel. $'% Most people hose average grocery bill is less than
1.

523 ould not be persuaded to spend more by any discount program. $)% Most people ho received discounts on home appliances through <ed @abels program ill shop at <ed @abel after the program ends. $*% Since the beginning of the discount program, most of the people ho spend 523 or more at <ed @abel are people ho have never before shopped there and hose average grocery bill has al ays been higher than 523. $+% Almost all of the people ho have begun spending 523 or more at <ed @abel since the discount program began are longtime customers ho have increased the average amount of their shopping bills by ma"ing fe er trips.

GMAT

7. Throughout the 1C23s, there ere increases in the numbers of dead birds found in agricultural areas after pesticide sprayings. 4esticide manufacturers claimed that the publicity given to bird deaths stimulated volunteers to loo" for dead birds, and that the increase in numbers reported as attributable to the increase in the number of people loo"ing. 8hich of the follo ing statements, if true, ould help to refute the claim of the pesticide manufacturers! $A%The publicity given to bird deaths as largely regional and never reached national proportions. $'% 4esticide sprayings ere timed to coincide ith various phases of the life cycles of the insects they destroyed. $)%No provision as made to ensure that a dead bird ould not be reported by more than one observer. $*% 1nitial increases in bird deaths had been noticed by agricultural or"ers long before any publicity had been given to the matter. $+% *ead birds of the same species as those found in agricultural areas had been found along coastal areas here no farming too" place. 6. Teenagers are often priced out of the labor mar"et by the government&mandated minimum& age level because employers cannot afford to pay that much for e/tra help. Therefore, if )ongress institutes a subminimum age, a ne lo er legal age for teenagers, the teenage unemployment rate, hich has been rising since 1C;3, ill no longer increase. 8hich of the follo ing, if true, ould most ea"en the argument above! $A% Since 1C;3 the teenage unemployment rate has risen hen the minimum age has risen. $'% Since 1C;3 the teenage unemployment rate has risen even hen the minimum age remained
10

constant. $)% +mployers often hire e/tra help during holiday and arm eather seasons. $*% The teenage unemployment rate rose more #uic"ly in the 1C73s than it did in the 1C;3s. $+% The teenage unemployment rate has occasionally declined in the years since 1C;3.

GMAT

C. 8hich of the follo ing best completes the passage belo ! The computer industrys estimate that it loses millions of dollars hen users illegally copy programs ithout paying for them is greatly e/aggerated. Most of the illegal copying is done by people ith no serious interest in the programs. Thus, the loss to the industry is much smaller than estimated because $A% many users ho illegally copy programs never find any use for them $'% most of the illegally copied programs ould not be purchased even if purchasing them ere the only ay to obtain them $)% even if the computer industry received all the revenue it claims to be losing, it ould still be e/periencing financial difficulties $*% the total mar"et value of all illegal copies is lo in comparison to the total revenue of the computer industry $+% the number of programs that are fre#uently copied illegally is lo in comparison to the number of programs available for sale 13. This year the Ne Iampshire *ivision of )ompany >, set a ne record for annual sales by that division. This record is especially surprising since the Ne Iampshire *ivision has the smallest potential mar"et and the lo est sales of any of )ompany >s divisions. 8hich of the follo ing identifies a fla in the logical coherence of the statement above! $A% 1f overall sales for )ompany > ere sharply reduced, the Ne Iampshire *ivisions ne sales record is irrelevant to the companys prosperity. $'% Since the division is competing against its o n record, the comparison of its sales record ith that
12

of other divisions is irrelevant. $)% 1f this is the first year that the Ne Iampshire *ivision has been last in sales among )ompany >s divisions, the ne record is not surprising at all. $*% 1f overall sales for )ompany > ere greater than usual, it is not surprising that the Ne Iampshire *ivision as last in sales. $+% Since the Ne Iampshire *ivision has the smallest potential mar"et, it is not surprising that it had the lo est sales.

GMAT

11. Statement of a :nited States copper mining company= 1mport #uotas should be imposed on the less e/pensive copper mined outside the country to maintain the price of copper in this countryE other ise, our companies ill not be able to stay in business. <esponse of a :nited States copper ire manufacturer= :nited States ire and cable manufacturers purchase about 73 percent of the copper mined in the :nited States. 1f the copper prices e pay are not at the international level, our sales ill drop, and then the demand for :nited States copper ill go do n. 1f the factual information presented by both companies is accurate, the best assessment of the logical relationship bet een the t o arguments is that the ire manufacturers argument $A% is self&serving and irrelevant to the proposal of the mining company $'% is circular, presupposing hat it see"s to prove about the proposal of the mining company $)% sho s that the proposal of the mining company ould have a negative effect on the mining companys o n business $*% fails to give a reason hy the proposal of the mining company should not be put into effect to alleviate the concern of the mining company for staying in business $+% establishes that even the mining companys business ill prosper if the mining companys proposal is re(ected 1-. J has been believed to cause K. A ne report, noting that J and K are often observed to be preceded by >, suggests that >, not J, may be the cause of K. 8hich of the follo ing further observations ould
1;

best support the ne reports suggestion! $A% 1n cases here > occurs but J does not, > is usually follo ed by K. $'% 1n cases here > occurs, follo ed by J, J is usually follo ed by K. $)% 1n cases here J occurs but > does not, J is usually follo ed by K. $*% 1n cases here J occurs but K does not, J is usually preceded by >. $+% 1n cases here K occurs, it is usually preceded by > and J.

GMAT

1.. Mr. 4rimm= 1f hospitals ere private enterprises, dependent on profits for their survival, there ould be no teaching hospitals, because of the intrinsically high cost of running such hospitals. Ms. Na"ai= 1 disagree. The medical challenges provided by teaching hospitals attract the very best physicians. This, in turn, enables those hospitals to concentrate on nonroutine cases. 8hich of the follo ing, if true, ould most strengthen Ms. Na"ais attempt to refute Mr. 4rimms claim! $A% *octors at teaching hospitals command high salaries. $'% Sophisticated, nonroutine medical care commands a high price. $)% +/isting teaching hospitals derive some revenue from public subsidies. $*% The patient mortality rate at teaching hospitals is high. $+% The modern trend among physicians is to become highly specialiBed. 10. A recent survey of all auto accident victims in *ole )ounty found that, of the severely in(ured drivers and front&seat passengers, 63 percent ere not earing seat belts at the time of their accidents. This indicates that, by earing seat belts, drivers and front&seat passengers can greatly reduce their ris" of being severely in(ured if they are in an auto accident. The conclusion above is not properly dra n unless hich of the follo ing is true! $A% ,f all the drivers and front&seat passengers in the survey, more than -3 percent ere earing seat belts at the time of their accidents. $'%)onsiderably more than -3 percent of drivers and front&seat passengers in *ole )ounty al ays ear
17

seat belts hen traveling by car. $)% More drivers and front& seat passengers in the survey than rear&seat passengers ere very severely in(ured. $*% More than half of the drivers and front&seat passengers in the survey ere not earing seat belts at the time of their accidents. $+% Most of the auto accidents reported to police in *ole )ounty do not involve any serious in(ury.

GMAT

12. Si/ months or so after getting a video recorder, many early buyers apparently lost interest in obtaining videos to atch on it. The trade of businesses selling and renting videos is still buoyant, because the number of homes ith video recorders is still gro ing. 'ut clearly, once the mar"et for video recorders is saturated, businesses distributing videos face hard times. 8hich of the follo ing, if true, ould most seriously ea"en the conclusion above! $A% The mar"et for video recorders ould not be considered saturated until there as one in 63 percent of homes. $'% Among the items handled by video distributors are many films specifically produced as video features. $)% 9e of the early buyers of video recorders raised any complaints about performance aspects of the ne product. $*% The early buyers of a novel product are al ays people ho are #uic" to ac#uire novelties, but also often as #uic" to tire of them. $+% 1n a shrin"ing mar"et, competition al ays intensifies and marginal businesses fail. 1;. Advertiser= The revenue that ne spapers and magaBines earn by publishing advertisements allo s publishers to "eep the prices per copy of their publications much lo er than ould other ise be possible. Therefore, consumers benefit economically from advertising. )onsumer= 'ut ho pays for the advertising that pays for lo &priced ne spapers and magaBines! 8e consumers do, because advertisers pass along advertising costs to us through the higher prices they charge for their products.

8hich of the follo ing best describes ho the consumer counters the advertisers argument! $A% 'y alleging something that, if true, ould ea"en the plausibility of the advertisers conclusion $'% 'y #uestioning the truth of the purportedly factual statement on hich the advertisers conclusion is based $)% 'y offering an interpretation of the advertisers opening statement that, if accurate, sho s that there is an implicit contradiction in it $*% 'y pointing out that the advertisers point of vie is biased $+% 'y arguing that the advertiser too narro ly restricts the discussion to the effects of advertising that are economic

16

GMAT

17. Mr. @a son= 8e should adopt a national family policy that includes legislation re#uiring employers to provide paid parental leave and establishing government&sponsored day care. Such la s ould decrease the stress levels of employees ho have responsibility for small children. Thus, such la s ould lead to happier, better&ad(usted families. 8hich of the follo ing, if true, ould most strengthen the conclusion above! $A% An employees high stress level can be a cause of unhappiness and poor ad(ustment for his or her family. $'% 4eople ho have responsibility for small children and ho or" outside the home have higher stress levels than those ho do not. $)% The goal of a national family policy is to lo er the stress levels of parents. $*% Any national family policy that is adopted ould include legislation re#uiring employers to provide paid parental leave and establishing government& sponsored day care. $+% Most children ho have been cared for in daycare centers are happy and ell ad(usted. 16. @ar" Manufacturing )ompany initiated a voluntary Luality )ircles program for machine operators. 1ndependent surveys of employee attitudes indicated that the machine operators participating in the program ere less satisfied ith their or" situations after t o years of the programs e/istence than they ere at the programs start. ,bviously, any or"ers ho participate in a Luality )ircles program ill, as a result, become less satisfied ith their (obs. +ach of the follo ing, if true, ould ea"en the conclusion dra n above +>)+T4= $A% The second survey occurred during a period of
1C

recession hen rumors of cutbac"s and layoffs at @ar" Manufacturing ere plentiful . $'% The surveys also sho ed that those @ar" machine operators ho neither participated in Luality )ircles nor "ne anyone ho did so reported the same degree of lessened satisfaction ith their or" situations as did the @ar" machine operators ho participated in Luality )ircles. $)% 8hile participating in Luality )ircles at @ar" Manufacturing, machine operators e/hibited t o of the primary indicators of improved (ob satisfaction= increased productivity and decreased absenteeism.

GMAT

$*% Several or"ers at @ar" Manufacturing ho had participated in Luality )ircles hile employed at other companies reported that, hile participating in Luality )ircles in their previous companies, their or" satisfaction had increased. $+% The machine operators ho participated in Luality )ircles reported that, hen the program started, they felt that participation might improve their or" situations. Luestions 1C&-3 are based on the follo ing. 'lood ban"s ill shortly start to screen all donors for NAN' hepatitis. Although the ne screening tests are estimated to dis#ualify up to 2 percent of all prospective blood donors, they ill still miss t o&thirds of donors carrying NAN' hepatitis. Therefore, about 13 percent of actual donors ill still supply NAN'&contaminated blood. 1C. The argument above depends on hich of the follo ing assumptions! $A% *onors carrying NAN' hepatitis do not, in a large percentage of cases, carry other infections for hich reliable screening tests are routinely performed. $'% *onors carrying NAN' hepatitis do not, in a large percentage of cases, develop the disease themselves at any point. $)% The estimate of the number of donors ho ould be dis#ualified by tests for NAN' hepatitis is an underestimate. $*% The incidence of NAN' hepatitis is lo er among the potential blood donors than it is in the population at large. $+% The donors ho ill still supply NAN'& contaminated blood ill donate blood at the average fre#uency for all donors. -3. 8hich of the follo ing inferences about the conse&
-3

#uences of instituting the ne tests is best supported by the passage above! $A% The incidence of ne cases of NAN' hepatitis is li"ely to go up by 13 percent. $'% *onations made by patients specifically for their o n use are li"ely to become less fre#uent. $)% The demand for blood from blood ban"s is li"ely to fluctuate more strongly. $*% The blood supplies available from blood ban"s are li"ely to go do n. $+% The number of prospective first&time donors is li"ely to go up by 2 percent.

GMAT

CRITICAL REASONING TEST SECTION 3 30 Minutes 20 Questions

1. )hilds 8orld, a chain of toy stores, has relied on a Gsupermar"et conceptH of computeriBed inventory control and customer self&service to eliminate the category of .. Millions of identical copies of a plant can be produced using sales cler"s from its force of employees. 1t no plans to ne tissue&culture and cloning employ the same concept in selling childrens clothes. techni#ues. The plan of )hilds 8orld assumes that $A% supermar"ets ill not also be selling childrens clothes in the same manner $'% personal service by sales personnel is not re#uired for selling childrens clothes successfully $)% the same "ind of computers ill be used in inventory control for both clothes and toys at )hilds 8orld $*% a self&service plan cannot be employed ithout computeriBed inventory control $+% sales cler"s are the only employees of )hilds 8orld ho could be assigned tas"s related to inventory control -. )ontinuous indoor fluorescent light benefits the health of hamsters ith inherited heart disease. A group of them e/posed to continuous fluorescent light survived t enty&five percent longer than a similar group e/posed instead to e#ual periods of indoor fluorescent light and of dar"ness. The method of the research described above is most li"ely to be applicable in addressing hich of the follo ing #uestions! $A% )an industrial or"ers ho need to see their or" do so better by sunlight or by fluorescent light! $'% )an hospital lighting be improved to promote the recovery of patients! $)% Io do deep&sea fish survive in total dar"ness! $*% 8hat are the inherited illnesses to hich hamsters are sub(ect!
-1

$+% Are there plants that re#uire specific periods of dar"ness in order to bloom!

1f plant propagation by such methods in laboratories proves economical, each of the follo ing, if true, represents a benefit of the ne techni#ues to farmers +>)+4T= $A% The techni#ues allo the development of superior strains to ta"e place more rapidly, re#uiring fe er generations of plants gro n to maturity.

GMAT

$'% 1t is less difficult to care for plants that ill gro at rates that do not vary idely. $)% 4lant diseases and pests, once they ta"e hold, spread more rapidly among genetically uniform plants than among those ith genetic variations. $*% Mechanical harvesting of crops is less difficult if plants are more uniform in siBe. $+% Special genetic traits can more easily be introduced into plant strains ith the use of the ne techni#ues. 0. 8hich of the follo ing best completes the passage belo ! Sales campaigns aimed at the faltering personal computer mar"et have strongly emphasiBed ease of use, called user&friendliness. This emphasis is oddly premature and irrelevant in the eyes of most potential buyers, ho are trying to address the logically prior issue of hether&&&& $A% user&friendliness also implies that o ners can service their o n computers $'% personal computers cost more the more user& friendly they are $)% currently available models are user&friendly enough to suit them $*% the people promoting personal computers use them in their o n homes $+% they have enough sensible uses for a personal computer to (ustify the e/pense of buying one 2. A eapons&smuggling incident recently too" place in country J. 8e all "no that J is a closed society. So Js government must have "no n about the eapons. 8hich of the follo ing is an assumption that ould ma"e the conclusion above logically correct! $A% 1f a government "no s about a particular eapons& smuggling incident, it must have intended to use
--

the eapons for its o n purposes. $'% 1f a government claims that it "ne nothing about a particular eapons& smuggling incident, it must have "no n everything about it. $)% 1f a government does not permit eapons to enter a country, it is a closed society. $*% 1f a country is a closed society, its government has a large contingent of armed guards patrolling its borders. $+% 1f a country is a closed society, its government has "no ledge about everything that occurs in the country.

GMAT

;. 'anning cigarette advertisements in the mass media ill not reduce the number of young people ho smo"e. They "no that cigarettes e/ist and they "no ho to get them. They do not need the advertisements to supply that information. The above argument ould be most ea"ened if hich of the follo ing ere true! $A% Seeing or hearing an advertisement for a product tends to increase peoples desire for that product. $'% 'anning cigarette advertisements in the mass media ill cause an increase in advertisements in places here cigarettes are sold. $)% Advertisements in the mass media have been an e/ceedingly large part of the e/penditures of the tobacco companies. $*% Those ho oppose cigarette use have advertised against it in the mass media ever since cigarettes ere found to be harmful. $+% ,lder people tend to be less influenced by mass& media advertisements than younger people tend to be. 7. 4eople tend to estimate the li"elihood of an events occurrence according to its salienceE that is, according to ho strongly and ho often it comes to their attention. 'y placement and headlines, ne spapers emphasiBe stories about local crime over stories about crime else here and about many other ma(or events. 1t can be concluded on the basis of the statements above that, if they are true, hich of the follo ing is most probably also true! $A% The language used in ne spaper headlines about local crime is inflammatory and fails to respect the rights of suspects. $'%The coverage of international events in ne spapers
-.

is neglected in favor of the coverage of local events. $)% <eaders of local ne s in ne spapers tend to overestimate the amount of crime in their o n localities relative to the amount of crime in other places. $*% None of the events concerning other people that are reported in ne spapers is so salient in peoples minds as their o n personal e/periences. $+% The press is the ne s medium that focuses peoples attention most strongly on local crimes.

GMAT

$+% an attac" on the character 6. 'y analyBing the garbage of a large number of average& of the opposition. siBed households, a group of modern urban anthropologists has found that a household discards 13. 8hich of the follo ing, if less food the more standardiBed made up of canned true, ould most li"ely be and prepac"aged foodsits diet is. The more part of the evidence used to standardiBed a households diet is, ho ever, the greater refute the conclusion above! the #uantities of fresh produce the household thro s $A% Accountants, la yers, a ay. and physicians attained 8hich of the follo ing can be properly inferred from their current relatively the passage! high levels of income and prestige at about the same $A% An increasing number of households rely on a time that the pay and highly standardiBed diet. status of teachers, ban" $'% The less standardiBed a households diet is, the tellers, and secretaries more nonfood aste the household discards. slipped. $)% The less standardiBed a households diet is, the $'% 8hen large numbers of smaller is the proportion of fresh produce in the men (oin a female& households food aste. dominated occupation, $*% The less standardiBed a households diet is, the such as airline flight more canned and prepac"aged foods the household attendant, the status and discards as aste. pay of the occupation $+% The more fresh produce a household buys, the more tend to increase. fresh produce it thro s a ay. Luestions CM13 are based on the follo ing. 1n the past, teachers, ban" tellers, and secretaries ere predominantly menE these occupations slipped in pay and status hen they became largely occupied by omen. Therefore, if omen become the ma(ority in currently male&dominated professions li"e accounting, la , and medicine, the income and prestige of these professions ill also drop. C. The argument above is based on $A% another argument that contains circular reasoning $'% an attempt to refute a generaliBation by means of an e/ceptional case $)% an analogy bet een the past and the future $*% an appeal to popular beliefs and values
-0

GMAT

1-. 8hich of the follo ing, if true, could best be used as a basis for arguing against the 11. An electric&po er company gained greater profits and authors position that the provided electricity to consumers at lo er rates per meteorologists claim unit of electricity by building larger&capacity more cannot be evaluated! efficient plants and by stimulating greater use of $A% )ertain unusual electricity ithin its area. To continue these financial configurations of data can trends, the company planned to replace an old plant serve as the basis for by a plant ith triple the capacity of its largest plant. precise eather forecasts The companys plan as described above assumed even though the e/act each of the follo ing +>)+4T= causal mechanisms are not understood. $A% *emand for electricity ithin the companys area of service ould increase in the future. $'% +/penses ould not rise beyond the level that could be compensated for by efficiency or volume of operation, or both. $)% The planned plant ould be sufficiently reliable in service to contribute a net financial benefit to the company as a hole. $*% Safety measures to be instituted for the ne plant ould be the same as those for the plant it ould replace. $+% The tripling of capacity ould not result in insuperable technological obstacles to efficiency. Luestions 1-&1. are based on the follo ing Meteorologists say that if only they could design an
-2

$)% The demand for teachers and secretaries has increased significantly in recent years, hile the demand for ban" tellers has remained relatively stable. $*% 1f present trends in the a arding of la degrees to omen continue, it ill be at least t o decades before the ma(ority of la yers are omen. $+% The pay and status of female accountants, la yers, and physicians today are governed by significantly different economic and sociological forces than ere the pay and status of female teachers, ban" tellers, and secretaries in the past.

accurate mathematical model of the atmosphere ith all its comple/ities, they could forecast the eather ith real precision. 'ut this is an idle boast, immune to any evaluation, for any inade#uate eather forecast ould obviously be blamed on imperfections in the model.

GMAT

$'% Most significant gains in the accuracy of the relevant mathematical models are accompanied by clear gains in the precision of eather forecasts. $)% Mathematical models of the meteorological aftermath of such catastrophic events as volcanic eruptions are beginning to be constructed. $*% Modern eather forecasts for as much as a full day ahead are broadly correct about 63 percent of the time. $+% Meteorologists readily concede that the accurate mathematical model they are tal"ing about is not no in their po er to construct. 1.. 8hich of the follo ing, if true, ould cast the most serious doubt on the meteorologists boast, aside from the doubt e/pressed in the passage above! $A% The amount of energy that the +arth receives from the Sun is monitored closely and is "no n not to be constant. $'% Folcanic eruptions, the combustion of fossil fuels, and several other processes that also cannot be #uantified ith any accuracy are "no n to have a significant and continuing impact on the constitution of the atmosphere. $)% As current models of the atmosphere are improved, even small increments in comple/ity ill mean large increases in the number of computers re#uired for the representation of the models. $*% 9re#uent and accurate data about the atmosphere collected at a large number of points both on and above the ground are a prere#uisite for the construction of a good model of the atmosphere. $+% 8ith e/isting models of the atmosphere, large scale eather patterns can be predicted ith greater accuracy than can relatively local eather patterns. 10. ,f the countries that ere the orlds t enty largest
-;

e/porters in 1C2., four had the same share of total orld e/ports in 1C60 as in 1C2.. Theses countries can therefore serve as models for those countries that ish to "eep their share of the global e/port trade stable over the years. 8hich of the follo ing, if true, casts the most serious doubt on the suitability of those four countries as models in the sense described! $A% Many countries ish to increase their share of orld e/port trade, not (ust "eep it stable. $'% Many countries are less concerned ith e/ports alone than ith he balance bet een e/ports and imports.

GMAT

$)% 8ith respect to the mi/ of products each e/ports, 1;. 8hich of the follo ing the four countries are very different from each #uestions can be ans ered other. on the basis of the $*% ,f the four countries, t o had a much larger, and information in the passage t o had a much smaller, share of total orld above! e/ports in 1C73 than in 1C60. $A% 1s the 4ostal Service as $+% The e/ports of the four countries range from 12 efficient as privately percent to 72 percent of the total national output. o ned electric transmission services! Luestions 12&1; are based on the follo ing $'% 1f private operators ere 1n the :nited States, the 4ostal Service has a monopoly allo ed to operate first& on first&class mail, but much of hat is sent first class class mail services, ould could be transmitted electronically. +lectronic transmittal they choose to do so! operators argue that if the 4ostal Service ere to offer $)% *o the electronic electronic transmission, it ould have an unfair transmittal operators advantage, since its electronic transmission service could believe that the 4ostal be subsidiBed from the profits of the monopoly. Service ma"es a profit on first&class mail! 12. 8hich of the follo ing, if each is true, ould allay $*% 1s the 4ostal Service the electronic transmittal operators fears of unfair prohibited from offering competition! electronic transmission $A% 1f the 4ostal Service ere to offer electronic services ! transmission, it could not ma"e a profit on first& $+% 1s the 4ostal Service class mail. e/pected to have a $'% 1f the 4ostal Service ere to offer electronic monopoly on electronic transmission, it ould have a monopoly on that transmission! "ind of service. $)% Much of the material that is no sent by first& class mail could be delivered much faster by special pac"age couriers, but is not sent that ay because of cost. $*% There is no economy of scale in electronic transmissionthat is, the cost per transaction does not go do n as more pieces of information are transmitted. $+% +lectronic transmission ill never be cost& effective for material not sent by first&class mail such as ne spapers and bul" mail.
-7

GMAT

17. @ists of hospitals have been compiled sho ing hich hospitals have patient death rates e/ceeding the national average. The data have been ad(usted to allo for differences in the ages of patients. +ach of the follo ing, if true, provides a good logical ground for hospitals to ob(ect to interpreting ran" on these lists as one of the indices of the #uality of hospital care +>)+4T= $A% <an" order might indicate insignificant differences, rather than large differences, in numbers of patient deaths. $'% Iospitals that "eep patients longer are li"ely to have higher death rates than those that discharge patients earlier but do not record deaths of patients at home after discharge. $)% 4atients ho are very old on admission to a hospital are less li"ely than younger patients to survive the same types of illnesses or surgical procedures. $*% Some hospitals serve a larger proportion of lo & income patients, ho tend to be more seriously ill hen admitted to a hospital. $+% 9or&profit hospitals sometimes do not provide intensive&care units and other e/pensive services for very sic" patients but refer or transfer such patients to other hospitals. 16. Teresa= Manned spaceflight does not have a future, since it cannot compete economically ith other means of accomplishing the ob(ectives of spaceflight. +d ard= No mode of human transportation has a better record of reliability= t o accidents in t enty& five years. Thus manned spaceflight definitely has a positive future. 8hich of the follo ing is the best logical evaluation of +d ards argument as a response to Teresas
-6

argument! $A% 1t cites evidence that, if true, tends to disprove the evidence cited by Teresa in dra ing her conclusion. $'% 1t indicates a logical gap in the support that Teresa offers for her conclusion. $)% 1t raises a consideration that out eighs the argument Teresa ma"es. $*% 1t does not meet Teresas point because it assumes that there is no serious impediment to transporting people into space, but this as the issue raised by Teresa. $+% 1t fails to respond to Teresas argument because it does not address the fundamental issue of hether space activities should have priority over other claims on the national budget.

GMAT

1C. 'lac" Americans are, on the hole, about t ice as li"ely as 8hite Americans to develop high blood pressure. This li"elihood also holds for esterniBed 'lac" Africans hen compared to 8hite Africans. <esearchers have hypothesiBed that this predisposition in esterniBed 'lac"s may reflect an interaction bet een estern high&salt diets and genes that adapted to an environmental scarcity of salt. 8hich of the follo ing statements about present&day, esterniBed 'lac" Africans, if true, ould most tend to confirm the researchers hypothesis! $A% The blood pressures of those descended from peoples situated throughout their history in Senegal and Gambia, here salt as al ays available, are lo . $'% The unusually high salt consumption in certain areas of Africa represents a serious health problem. $)% 'ecause of their blood pressure levels, most 8hite Africans have mar"edly decreased their salt consumption. $*% 'lood pressures are lo among the Joruba, ho, throughout their history, have been situated far inland from sources of sea salt and far south of Saharan salt mines. $+% No significant differences in salt metabolism have been found bet een those people ho have had salt available throughout their history and those ho have not.

-3. The follo ing proposal to amend the byla s of an organiBation as circulated to its members for comment. 8hen more than one nominee is to be named for an office, prospective nominees must consent to nomination and before giving such consent must be told ho the other nominees ill be. 8hich of the follo ing comments concerning the logic of the proposal is accurate if it cannot be "no n ho the actual nominees are until prospective nominees have given their consent to be nominated! $A% The proposal ould ma"e it possible for each of several nominees for an office to be a are of ho all of the other nominees are. $'% The proposal ould iden the choice available to those choosing among the nominees. $)% 1f there are several prospective nominees, the proposal ould deny the last nominee e#ual

-C

GMAT

treatment ith the first. $*%The proposal ould enable a prospective nominee to ithdra from competition ith a specific person ithout ma"ing that ithdra al "no n. $+% 1f there is more than one prospective nominee, the proposal ould ma"e it impossible for anyone to become a nominee.

CRITICAL REASONING TEST SECTION 4 30 Minutes 20 Questions 1. 8hich of the follo ing best completes the passage belo ! 1n a survey of (ob applicants, t o&fifths admitted to being at least a little dishonest. Io ever, the survey may underestimate the proportion of (ob applicants ho are dishonest, because. $A% some dishonest people ta"ing the survey might have claimed on the survey to be honest $'% some generally honest people ta"ing the survey might have claimed on the survey to be dishonest $)% some people ho claimed on the survey to be at least a little dishonest may be very dishonest $*% some people ho claimed on the survey to be dishonest may have been ans ering honestly $+% some people ho are not (ob applicants are probably at least a little dishonest Luestions -&. are based on the follo ing. The average life e/pectancy for the :nited States population as a

.3

GMAT

hole is 7..C years, but children born in Ia aii ill live an average of 77 years, and those born in @ouisiana, 71.7 years. 1f a ne ly ed couple from @ouisiana ere to begin their family in Ia aii, therefore, their children ould be e/pected to live longer than ould be the case if the family remained in @ouisiana. -. 8hich of the follo ing, if true, ould most seriously ea"en the conclusion dra n in the passage! $A% 1nsurance company statisticians do not believe that moving to Ia aii ill significantly lengthen the average @ouisianians life. $'% The governor of @ouisiana has falsely alleged that statistics for his state are inaccurate. $)% The longevity ascribed to Ia aiis current population is attributable mostly to genetically determined factors. $*% Thirty percent of all @ouisianians can e/pect to live longer than 77 years. $+% Most of the Ia aiian 1slands have levels of air pollution ell belo the national average for the :nited States.

.. 8hich of the follo ing statements, if true, ould most significantly strengthen the conclusion dra n in the passage! $A% As population density increases in Ia aii, life e/pectancy figures for that state are li"ely to be revised do n ard. $'% +nvironmental factors tending to favor longevity are abundant in Ia aii and less numerous in @ouisiana. $)% T enty&five percent of all @ouisianians ho move to Ia aii live longer than 77 years. $*% ,ver the last decade, average life e/pectancy has risen at a higher rate for @ouisianians than for Ia aiians. $+% Studies sho that the average life e/pectancy for Ia aiians ho move permanently to @ouisiana is roughly e#ual to that of Ia aiians ho remain in Ia aii. 0. 1nsurance )ompany > is considering issuing a ne policy to cover services re#uired by elderly people ho suffer from diseases that afflict the elderly. 4remiums

.1

GMAT

2. A program instituted in a particular state allo s parents to prepay their childrens future college tuition at current 8hich of the follo ing strategies ould be most li"ely rates. The program then pays to minimiBe )ompany >s losses on the policies! the tuition annually for the $A% Attracting middle&aged customers unli"ely to child at any of the states submit claims for benefits for many years public colleges in hich the $'% 1nsuring only those individuals ho did not suffer child enrolls. 4arents should any serious diseases as children participate in the program as a $)% 1ncluding a greater number of services in the policy means of decreasing the cost than are included in other policies of lo er cost for their childrens college $*% 1nsuring only those individuals ho ere re(ected education. by other companies for similar policies 8hich of the follo ing, if $+% 1nsuring only those individuals ho are ealthy true, is the most appropriate enough to pay for the medical services reason for parents not to participate in the program! $A% The parents are unsure about hich pubic college in the state the child ill attend. $'% The amount of money accumulated by putting the prepayment funds in an interest&bearing account today ill be greater than the total cost of tuition for any of the pubic colleges hen the child enrolls. $)% The annual cost of tuition at the states pubic colleges is e/pected to increase at a faster rate than the annual increase in the cost of living $*% Some of the states public
.-

for the policy must be lo enough to attract customers. Therefore, )ompany > is concerned that the income from the policies ould not be sufficient to pay for the claims that ould be made.

GMAT

colleges are contemplating large increases in tuition ne/t year. $+% The prepayment plan ould not cover the cost of room and board at any of the states public colleges. ;. )ompany Alpha buys free&travel coupons from people ho are a arded the coupons by 'ravo Airlines for flying fre#uently on 'ravo airplanes. The coupons are sold to people ho pay less for the coupons than they ould pay by purchasing tic"ets from 'ravo. This mar"eting of coupons results in lost revenue for 'ravo. To discourage the buying and selling of free&travel coupons, it ould be best for 'ravo Airlines to restrict the $A% number of coupons that a person can be a arded in a particular year $'% use of the coupons to those ho ere a arded the coupons and members of their immediate families $)% days that the coupons can be used to Monday through 9riday $*% amount of time that the coupons can be used after they are issued $+% number of routes on hich travelers can use the coupons

7. The ice on the front indshield of the car had formed hen moisture condensed during the night. The ice melted #uic"ly after the car as armed up the ne/t morning because the defrosting vent, hich blo s only on the front indshield, as turned on full force. 8hich of the follo ing, if true, most seriously (eopardiBes the validity of the e/planation for the speed ith hich the ice melted! $A% The side indo s had no ice condensation on them. $'% +ven though no attempt as made to defrost the bac" indo , the ice there melted at the same rate as did the ice on the front indshield. $)% The speed at hich ice on a indo melts increases as the temperature of the air blo n on the indo increases. $*% The arm air from the defrosting vent for the front indshield cools rapidly as it dissipates throughout the rest of the car. $+% The defrosting vent operates efficiently even hen the heater, hich

..

GMAT

C. A conservation group in the :nited States is trying to 6. To prevent some conflicts of interest, )ongress could change the long&standing prohibit high&level government officials from accepting image of bats as frightening positions as lobbyists for three years after such officials creatures. The group contends leave government service. ,ne such official concluded, that bats are feared and ho ever, that such a prohibition ould be unfortunate persecuted solely because they because it ould prevent high&level government are shy animals that are active officials from earning a livelihood for three years. only at night. The officials conclusion logically depends on hich of the follo ing assumptions! $A% @a s should not restrict the behavior of former government officials. $'% @obbyists are typically people ho have previously been high&level government officials. $)% @o &level government officials do not often become lobbyists hen they leave government service. $*% Iigh&level government officials ho leave government service are capable of earning a livelihood only as lobbyists. $+% Iigh&level government officials ho leave government service are currently permitted to act as lobbyists for only three years. 8hich of the follo ing, if true, ould cast the most serious doubt on the accuracy of the groups contention! $A% 'ats are steadily losing natural roosting places such as caves and hollo trees and are thus turning to more developed areas for roosting. $'% 'ats are the chief consumers of nocturnal insects and thus can help ma"e their hunting territory more pleasant for humans. $)% 'ats are regarded as frightening creatures not only in the :nited States but also in +urope, Africa, and South America. $*% <accoons and o ls are shy and active only at nightE yet they are not generally feared and persecuted. $+% 4eople "no more about

blo s arm air to ard the feet or faces of the driver and passengers, is on.

.0

GMAT

Luestions 11&1- are based on the follo ing. The fe er restrictions there are on the advertising of legal 13. Meteorite e/plosions in the +arths atmosphere as services, the more la yers there large as the one that destroyed forests in Siberia, ith are ho advertise their services, appro/imately the force of a t elve&megaton nuclear and the la yers ho advertise a blast, occur about once a century. specific service usually charge The response of highly automated systems controlled less for that service than la yers ho do not advertise. Therefore, by comple/ computer programs to une/pected if the state removes any of its circumstances is unpredictable. current restrictions, such as the 8hich of the follo ing conclusions can most one against advertisements that properly be dra n, if the statements above are true, do not specify fee arrangements, about a highly automated nuclear&missile defense overall consumer legal costs ill system controlled by a comple/ computer program! be lo er than if the state retains $A% 8ithin a century after its construction, the its current restrictions. system ould react inappropriately and might 11. 1f the statements above are accidentally start a nuclear ar. true, hich of the follo ing $'% The system ould be destroyed if an e/plosion must be true! of a large meteorite occurred in the +arths atmosphere. $A% Some la yers ho no $)% 1t ould be impossible for the system to advertise ill charge distinguish the e/plosion of a large meteorite from more for specific services the e/plosion of a nuclear eapon. if they do not have to $*% 8hether the system ould respond specify fee arrangements inappropriately to the e/plosion of a large in the advertisements. meteorite ould depend on the location of the $'% More consumers ill blast. use legal services if there $+% 1t is not certain hat the systems response to the are fe er restrictions on e/plosion of a large meteorite ould be, if its the advertising of legal designers did not plan for such a contingency. services. $)% 1f the restriction against advertisements that do not specify fee arrangements is removed, more la yers ill advertise their services.
.2

the behavior of other greatly feared animal species, such as lions, alligators, and sna"es, than they do about the behavior of bats.

GMAT

1.. *efense *epartment analysts orry that the ability of the :nited States to age a prolonged ar ould be seriously endangered if the machine&tool manufacturing base shrin"s further. 'efore the *efense *epartment publicly connected this 1-. 8hich of the follo ing, if true, ould most seriously security issue ith the ea"en the argument concerning overall consumer import #uota issue, legal costs! ho ever, the machine&tool industry raised the national $A% The state has recently removed some other security issue in its petition restrictions that had limited the advertising of legal for import #uotas. services. $'% The state is unli"ely to remove all of the restrictions that apply solely to the advertising of legal services. $)% @a yers ho do not advertise generally provide legal services of the same #uality as those provided by la yers ho do advertise. $*% Most la yers ho no specify fee arrangements in their advertisements ould continue to do so even if the specification ere not re#uired. $+% Most la yers ho advertise specific services do not lo er their fees for those services hen they begin to advertise. 8hich of the follo ing, if true, contributes most to an e/planation of the machine& tool industrys raising the issue above regarding national security! $A% 8hen the aircraft industries retooled, they provided a large amount of or" for tool builders. $'% The *efense *epartment is only marginally concerned ith the effects of foreign competition on the machine&tool industry. $)% The machine&tool industry encountered difficulty in obtaining governmental protection against imports on grounds other than defense.

$*% 1f more la yers advertise lo er prices for specific services, some la yers ho do not advertise ill also charge less than they currently charge for those services. $+% 1f the only restrictions on the advertising of legal services ere those that apply to every type of advertising, most la yers ould advertise their services.

.;

GMAT

$*% A fe eapons important for defense consist of 12. The cost of producing radios parts that do not re#uire e/tensive machining. in )ountry L is ten percent $+% Several federal government programs have been less than the cost of designed hich ill enable domestic machine&tool producing radios in )ountry manufacturing firms to compete successfully ith J. +ven after transportation foreign toolma"ers. fees and tariff charges are 10. ,pponents of la s that re#uire automobile drivers and added, it is still cheaper for passengers to ear seat belts argue that in a free a company to import radios society people have the right to ta"e ris"s as long as from )ountry L to )ountry the people do not harm others as a result of ta"ing J than to produce radios in the ris"s. As a result, they conclude that it should be )ountry J. each persons decision hether or not to ear a seat The statements above, if belt. true, best support hich of 8hich of the follo ing, if true, most seriously the follo ing assertions! ea"ens the conclusion dra n above! $A% @abor costs in )ountry $A% Many ne cars are built ith seat belts that L are ten percent belo automatically fasten hen someone sits in the those in )ountry J. front seat. $'% 1mporting radios from $'% Automobile insurance rates for all automobile )ountry L to )ountry J o ners are higher because of the need to pay for ill eliminate ten percent the increased in(uries or deaths of people not of the manufacturing (obs earing seat belts. in )ountry J. $)% 4assengers in airplanes are re#uired to ear seat $)% The tariff on a radio belts during ta"eoffs and landings. imported from )ountry L $*% The rate of automobile fatalities in states that do to )ountry J is less than not have mandatory seat&belt la s is greater than ten percent of the cost of the rate of fatalities in states that do have such manufacturing the radio la s. in )ountry J. $+% 1n automobile accidents, a greater number of $*% The fee for transporting passengers ho do not ear seat belts are in(ured a radio from )ountry L to than are passengers ho do ear seat belts. )ountry J is more than ten percent of the cost of manufacturing the radio in )ountry L. $+% 1t ta"es ten percent less time to manufacture a radio in )ountry L than it
.7

GMAT

does in )ountry J. 17. ,ne state adds a 7 percent 1;. *uring the Second 8orld 8ar, about .72,333 sales ta/ to the price of most civilians died in the :nited States and about 036,333 products purchased ithin members of the :nited States armed forces died its (urisdiction. This ta/, overseas. ,n the basis of those figures, it can be therefore, if vie ed as ta/ concluded that it as not much more dangerous to be on income, has the reverse overseas in the armed forces during the Second effect of the federal income 8orld 8ar than it as to stay at home as a civilian. ta/= the lo er the income, the higher the annual 8hich of the follo ing ould reveal most clearly percentage rate at hich the the absurdity of the conclusion dra n above! income is ta/ed. $A% )ounting deaths among members of the armed The conclusion above forces ho served in the :nited States in addition ould be properly dra n if to deaths among members of the armed forces hich of the follo ing ere serving overseas assumed as a premise! $'% +/pressing the difference bet een the numbers of deaths among civilians and members of the $A% The amount of money armed forces as a percentage of the total number citiBens spend on of deaths products sub(ect to the $)% Separating deaths caused by accidents during state ta/ tends to be e#ual service in the armed forces from deaths caused by across income levels. combat in(uries $'% The federal income ta/ $*% )omparing death rates per thousand members of favors citiBens ith high each group rather than comparing total numbers of incomes, hereas the deaths state sales ta/ favors $+% )omparing deaths caused by accidents in the citiBens ith lo :nited States to deaths caused by combat in the incomes. armed forces. $)% )itiBens ith lo annual incomes can afford to pay a relatively higher percentage of their incomes in state sales ta/, since their federal income ta/ is relatively lo . $*% The lo er a states sales ta/, the more it ill tend to redistribute income from the more affluent
.6

GMAT

citiBens to the rest of society. $+% )itiBens ho fail to earn federally ta/able income are also e/empt from the state sales ta/.

16. The average age of chief e/ecutive officers $)+,s% in a large sample of companies is 27. The average age of )+,s in those same companies -3 years ago as appro/imately eight years younger. ,n the basis of those data, it can be concluded that )+,s in general tend to be older no . 8hich of the follo ing casts the most doubt on the conclusion dra n above! $A% The dates hen the )+,s assumed their current positions have not been specified. $'% No information is given concerning the average number of years that )+,s remain in office. $)% The information is based only on companies that have been operating for at least -3 years. $*% ,nly appro/imate information is given concerning the average age of the )+,s -3 years ago. $+% 1nformation concerning the e/act number of companies in the sample has not been given.

Luestions 1C&-3 are based on the follo ing. Surveys sho that every year only 13 percent of cigarette smo"ers s itch brands. Jet the manufacturers have been spending an amount e#ual to 13 percent of their gross receipts on cigarette promotion in magaBines. 1t follo s from these figures that inducing cigarette smo"ers to s itch brands did not pay, and that cigarette companies ould have been no orse off economically if they had dropped their advertising. 1C. ,f the follo ing, the best criticism of the conclusion that inducing cigarette smo"ers to s itch brands did not pay is that the conclusion is based on $A% computing advertising costs as a percentage of gross receipts, not of overall costs $'% past patterns of smo"ing and may not carry over to the future $)% the assumption that each smo"er is loyal to a single brand of cigarettes at any one time $*% the assumption that each manufacturer produces only one brand of cigarettes

.C

GMAT

$+% figures for the cigarette industry as a hole and may not hold for a particular company -3. 8hich of the follo ing, if true, most serinously ea"ens the conclusion that cigarette companies could have dropped advertising ithout suffering economically!

CRITICAL REASONING TEST SECTION 5 30 MINUTES 20 QUESTIONS

1. Toughened hiring standards have not been the primary cause of the present staffing $A% )igarette advertisements provide a ma(or shortage in public schools. proportion of total advertising revenue for The shortage of teachers is numerous magaBines. primarily caused by the fact $'% )igarette promotion serves to attract first&time that in recent years teachers smo"ers to replace those people ho have stopped have not e/perienced any smo"ing. improvements in or"ing $)% There e/ists no research conclusively demon& conditions and their salaries strating that increases in cigarette advertising are have not "ept pace ith related to increases in smo"ing. salaries in other professions. $*% Advertising is so firmly established as a ma(or business activity of cigarette manufacturers that 8hich of the follo ing, if they ould be unli"ely to drop it. true, ould most support the $+% 'rand loyalty is typically not very strong among claims above! those ho smo"e ine/pensive cigarettes. $A% Many teachers already in the profession ould not have been hired under the ne hiring standards. $'% Today more teachers are entering the profession ith a higher educational level than in the past. $)% Some teachers have cited higher standards for hiring as a reason for the current staffing shortage. $*% Many teachers have cited lo pay and lac" of professional freedom as reasons for their leaving the profession. $+% Many prospective teachers
03

GMAT

have cited the ne hiring standards as a reason for .. +ven though most universities not entering the profession. retain the royalties from -. A proposed ordinance re#uires the installation in ne faculty members inventions, homes of sprin"lers automatically triggered by the the faculty members retain the presence of a fire. Io ever, a home builder argued that royalties from boo"s and because more than ninety percent of residential fires are articles they rite. Therefore, e/tinguished by a household member, residential faculty members should retain sprin"lers ould only marginally decrease property the royalties from the damage caused by residential fires. educational computer soft are they develop. 8hich of the follo ing, if true, ould most seriously ea"en the home builders argument! The conclusion above ould be more reasonably dra n if $A% Most individuals have no formal training in ho to hich of the follo ing ere e/tinguish fires. inserted into the argument as $'% Since ne homes are only a tiny percentage of an additional premise! available housing in the city, the ne ordinance ould be e/tremely narro in scope. $A% <oyalties from inventions $)% The installation of smo"e detectors in ne are higher than royalties residences costs significantly less than the from educational soft are installation of sprin"lers. programs. $*% 1n the city here the ordinance as proposed, the $'% 9aculty members are more average time re#uired by the fire department to li"ely to produce respond to a fire as less than the national educational soft are average. programs than inventions. $+% The largest proportion of property damage that $)% 1nventions bring more results from residential fires is caused by fires that prestige to universities start hen no household member is present. than do boo"s and articles. $*% 1n the e/perience of most universities, educational soft are programs are more mar"etable than are boo"s and articles. $+% 1n terms of the criteria used to a ard royalties, educational soft are programs are more nearly comparable to boo"s and
01

GMAT

articles than to inventions. 0. 1ncreases in the level of high&density lipoprotein $I*@% in the human bloodstream lo er bloodstream& cholesterol levels by increasing the bodys capacity to rid itself of e/cess cholesterol. @evels of I*@ in the bloodstream of some individuals are significantly increased by a program of regular e/ercise and eight reduction. 8hich of the follo ing can be correctly inferred from the statements above! $A% 1ndividuals ho are under eight do not run any ris" of developing high levels of cholesterol in the bloodstream. $'% 1ndividuals ho do not e/ercise regularly have a high ris" of developing high levels of cholesterol in the bloodstream late in life. $)% +/ercise and eight reduction are the most effective methods of lo ering bloodstream cholesterol levels in humans. $*% A program of regular e/ercise and eight reduction lo ers cholesterol levels in the bloodstream of some individuals. $+% ,nly regular e/ercise is necessary to decrease cholesterol levels in the bloodstream of individuals of average eight. 2. 8hen limitations ere in effect on nuclear&arms testing, people tended to save more of their money, but hen nuclear& arms testing increased, people tended to spend more of their money. The perceived threat of nuclear catastrophe, therefore, decreases the illingness of people to postpone consumption for the sa"e of saving money. The argument above assumes that $A% the perceived threat of nuclear catastrophe has increased over the years. $'% most people supported the development of nuclear arms $)% peoples perception of the threat of nuclear catastrophe depends on the amount of nuclear& arms testing being done $*% the people ho saved the most money hen nuclear&arms testing as limited ere the ones ho supported such limitations $+% there are more consumer goods available hen nuclear&arms testing increases

0-

GMAT

;. 8hich of the follo ing best completes the passage belo ! 4eople buy prestige hen they buy a premium product. They ant to be associated ith something special. Mass&mar"eting techni#ues and price&reduction strategies should not be used because AAAAAAA. $A% affluent purchasers currently represent a shrin"ing portion of the population of all purchasers $'% continued sales depend directly on the maintenance of an aura of e/clusivity $)% purchasers of premium products are concerned ith the #uality as ell as ith the price of the products $*% e/pansion of the mar"et niche to include a broader spectrum of consumers ill increase profits $+% manufacturing a premium brand is not necessarily more costly than manufacturing a standard brand of the same product

7. A cost&effective solution to the problem of airport congestion is to provide high&speed ground transportation bet een ma(or cities lying -33 to 233 miles apart. The successful implementation of this plan ould cost far less than e/panding e/isting airports and ould also reduce the number of airplanes clogging both airports and air ays. 8hich of the follo ing, if true, could proponents of the plan above most appropriately cite as a piece of evidence for the soundness of their plan! $A%An effective high&speed ground&transportation system ould re#uire ma(or repairs to many high ays and mass& transit improvements. $'% ,ne&half of all departing flights in the nations busiest airport head for a destination in a ma(or city --2 miles a ay. $)% The ma(ority of travelers departing from rural airports are flying to destinations in cities over ;33 miles a ay. $*% Many ne airports are being built in areas that are presently served by high&speed ground&

0.

GMAT

transportation systems. $+% A large proportion of air travelers are vacationers ho are ta"ing long&distance flights. Luestions 6&C are based on the follo ing. 1f there is an oil&supply disruption resulting in higher international oil prices, domestic oil prices in open& mar"et countries such as the :nited States ill rise as ell, hether such countries import all or none of their oil. 6. 1f the statement above concerning oil&supply disruptions is true, hich of the follo ing policies in an open&mar"et nation is most li"ely to reduce the long& term economic impact on that nation of sharp and une/pected increases in international oil prices! $A% Maintaining the #uantity of oil imported at constant yearly levels $'% 1ncreasing the number of oil tan"ers in its fleet $)% Suspending diplomatic relations ith ma(or oil& producing nations $*% *ecreasing oil consumption through conservation $+% *ecreasing domestic production of oil

C. 8hich of the follo ing conclusions is best supported by the statement above! $A% *omestic producers of oil in open&mar"et countries are e/cluded from the international oil mar"et hen there is a disruption in the international oil supply. $'% 1nternational oil&supply disruptions have little, if any, effect on the price of domestic oil as long as an open&mar"et country has domestic supplies capable of meeting domestic demand. $)% The oil mar"et in an open& mar"et country is actually part of the international oil mar"et, even if most of that countrys domestic oil is usually sold to consumers ithin its borders. $*% ,pen&mar"et countries that e/port little or none of their oil can maintain stable domestic oil prices even hen international oil prices rise sharply. $+% 1f international oil prices rise, domestic distributors of oil in open&mar"et countries ill begin to import more oil than they

00

GMAT

e/port. 13. The average normal infant born in the :nited States eighs bet een t elve and fourteen pounds at the age of three months. Therefore, if a three&month&old child eighs only ten pounds, its eight gain has been belo the :nited States average. 8hich of the follo ing indicates a fla in the reasoning above! $A% 8eight is only one measure of normal infant development. $'% Some three&month&old children eigh as much as seventeen pounds. $)% 1t is possible for a normal child to eigh ten pounds at birth. $*% The phrase G belo averageH does not necessarily mean insufficient. $+%Average eight gain is not the same as average eight.

11. <ed blood cells in hich the malarial&fever parasite resides are eliminated from a persons body after 1-3 days. 'ecause the parasite cannot travel to a ne generation of red blood cells, any fever that develops in a person more than 1-3 days after that person has moved to a malaria&free region is not due to the malarial parasite. 8hich of the follo ing, if true, most seriously ea"ens the conclusion above! $A% The fever caused by the malarial parasite may resemble the fever caused by flu viruses. $'% The anopheles mos#uito, hich is the principal insect carrier of the malarial parasite, has been eradicated in many parts of the orld. $)% Many malarial symptoms other than the fever, hich can be suppressed ith antimalarial medication, can reappear ithin 1-3 days after the medication is discontinued. $*% 1n some cases, the parasite that causes malarial fever travels to

02

GMAT

cells of the spleen, hich are less fre#uently eliminated from a persons body than are red blood cells. $+% 1n any region infested ith malaria&carrying mos#uitoes, there are individuals ho appear to be immune to malaria. 1-. 9act 1= Television advertising is becoming less effective= the proportion of brand names promoted on television that vie ers of the advertising can recall is slo ly decreasing. 9act -= Television vie ers recall commercials aired first or last in a cluster of consecutive commercials far better than they recall commercials aired some here in the middle. 9act - ould be most li"ely to contribute to an e/planation of fact 1 if hich of the follo ing ere also true! $A% The average television vie er currently recalls fe er than half the brand names promoted in commercials he or she sa . $'% The total time allotted to the average cluster of consecutive television commercials is decreasing. $)% The average number of hours per day that people spend atching television is decreasing. $*% The average number of clusters of consecutive commercials per hour of television is increasing. $+% The average number of television commercials in a cluster of consecutive commercials is increasing.

1.. The number of people diagnosed as having a certain intestinal disease has dropped significantly in a rural county this year, as compared to last year, Iealth officials attribute this decrease entirely to improved sanitary conditions at ater& treatment plants, hich made for cleaner ater this year and thus reduced the incidence of the disease. 8hich of the follo ing, if true, ould most seriously ea"en the health officials e/planation for the lo er incidence of the disease! $A% Many ne ater& treatment plants have been built in the last five years in the rural county. $'% 'ottled spring ater has not been consumed in significantly different #uantities by people diagnosed as having the intestinal disease, as compared to people ho did not contract the disease. $)% 'ecause of a ne diagnostic techni#ue, many people ho until this year ould have been diagnosed as having the intestinal disease are no

0;

GMAT

correctly diagnosed as suffering from intestinal 12. Some ho favor putting ulcers. governmental enterprises $*% 'ecause of medical advances this year, far fe er into private hands suggest people ho contract the intestinal disease ill that conservation ob(ectives develop severe cases of the disease. ould in general be better $+% The ater in the rural county as brought up to served if private the sanitary standards of the ater in neighboring environmental groups ere counties ten years ago. put in charge of operating and financing the national 10. The price the government pays for standard eapons par" system, hich is no purchased from military contractors is determined by run by the government. a pricing method called Ghistorical costing.H 8hich of the follo ing, Iistorical costing allo s contractors to protect their assuming that it is a realistic profits by adding a percentage increase, based on the possibility, argues most current rate of inflation, to the previous years strongly against the contractual price. suggestion above! 8hich of the follo ing statements, if true, is the best $A% Those see"ing to basis for a criticism of historical costing as an abolish all restrictions on economically sound pricing method for military e/ploiting the natural contracts! resources of the par"s $A% The government might continue to pay for past might (oin the private inefficient use of funds. environmental groups as $'% The rate of inflation has varied considerably over members and eventually the past t enty years. ta"e over their leadership. $)% The contractual price ill be greatly affected by $'% 4rivate environmental the cost of materials used for the products. groups might not al ays $*% Many ta/payers #uestion the amount of money agree on the best ays to the government spends on military contracts. achieve conservation $+% The pricing method based on historical costing ob(ectives. might not encourage the development of $)% 1f they ished to e/tend innovative eapons. the par" system, the private environmental groups might have to see" contributions from ma(or donors and the general public. $*% There might be competition among
07

GMAT

private environmental groups for control of certain 17. Toc#ueville, a nineteenth& par" areas. century riter "no n for his $+% Some endangered species, such as the )alifornia study of democracy in the condor, might die out despite the best efforts of the :nited States, believed that private environmental groups, even if those groups a government that are not hampered by insufficient resources. centraliBes po er in one 1;. A recent spate of launching and operating mishaps individual or institution is ith television satellites led to a corresponding surge dangerous to its citiBens. in claims against companies under riting satellite 'iographers claim that insurance. As a result, insurance premiums shot up, Toc#ueville disli"ed& ma"ing satellites more e/pensive to launch and centraliBed government operate. This, in turn, has added to the pressure to because he blamed s#ueeBe more performance out of currently operating Napoleons rule for the satellites. poverty of his childhood in Normandy. 8hich of the follo ing, if true, ta"en together ith the information above, best supports the conclusion 8hich of the follo ing, if that the cost of television satellites ill continue to true, ould cast the most increase! serious doubt on the biographers claim! $A% Since the ris" to insurers of satellites is spread over relatively fe units, insurance premiums are $A% Although Napoleon as necessarily very high. popularly blamed at the $'% 8hen satellites reach orbit and then fail, the time for the terrible living causes of failure are generally impossible to conditions in Normandy, pinpoint ith confidence. historians no "no that $)% The greater the performance demands placed on bad harvests ere really satellites, the more fre#uently those satellites brea" to blame for the poor do n. economic conditions. $*% Most satellites are produced in such small $'% Napoleon as notorious numbers that no economies of scale can be for refusing to share realiBed. po er ith any of his $+% Since many satellites are built by un ieldy political associates. international consortia, inefficiencies are $)% Toc#ueville said he inevitable. "ne that if his father had not suffered ill health, his family ould have had a steady income and a comfortable standard of
06

GMAT

living. $*% Although Toc#ueville asserted that :nited States 1C. <ecent estimates predict that bet een 1C6- and 1CC2 the political life as democratic, the :nited States of greatest increase in the the nineteenth century allo ed political po er to number of people employed be concentrated in a fe institutions. ill be in the category of $+% Toc#ueville once rote in a letter that, although lo &paying service his childhood as terribly impoverished, it as occupations. This category, not different from the e/perience of his friends and ho ever, ill not increase neighbors in Normandy. its share of total 16. <adio interferometry is a techni#ue for studying employment, hereas the details of celestial ob(ects that combines signals category of high&paying intercepted by idely spaced radio telescopes. This service occupations ill techni#ue re#uires ultraprecise timing, e/act increase its share. "no ledge of the locations of the telescopes, and 1f the estimates above are sophisticated computer programs. The successful accurate, hich of the interferometric lin"ing of an +arth&based radio follo ing conclusions can telescope ith a radio telescope on an orbiting be dra n! satellite as therefore a significant technological $A% 1n 1C6- more people accomplishment. ere or"ing in lo & 8hich of the follo ing can be correctly inferred paying service from the statements above! occupations than ere $A% Special care as ta"en in the launching of the or"ing in high&paying satellite so that the calculations of its orbit ould service occupations. be facilitated. $'% 1n 1CC2 more people $'% The signals received on the satellite are stronger ill be or"ing in high& than those received by a terrestrial telescope. paying service $)% The resolution of detail achieved by the satellite& occupations than ill be +arth interferometer system is inferior to that or"ing in lo &paying achieved by e/clusively terrestrial systems. service occupations. $*% The computer programs re#uired for ma"ing use $)% Nonservice occupations of the signals received by the satellite re#uired a ill account for the same long time for development. share of total employment $+% The location of an orbiting satellite relative to in 1CC2 as in 1C6-. locations on +arth can be ell enough "no n for $*% Many of the people ho interferometric purposes. ere or"ing in lo & paying service occupations in 1C6- ill
0C

GMAT

be or"ing in high&paying service occupations by 1CC2. $+% The rate of gro th for lo &paying service occupations ill be greater than the overall rate of employment gro th bet een 1C6- and 1CC2. -3. 9or a local government to outla all stri"es by its or"ers is a costly mista"e, because all its labor disputes must then be settled by binding arbitration, ithout any negotiated public&sector labor settlements guiding the arbitrators. Stri"es should be outla ed only for categories of public&sector or"ers for hose services no acceptable substitute e/ists. The statements above best support hich of the follo ing conclusions! $A% 8here public&service or"ers are permitted to stri"e, contract negotiations ith those or"ers are typically settled ithout a stri"e. $'% 8here stri"es by all categories of pubic&sector or"ers are outla ed, no acceptable substitutes for the services provided by any of those or"ers are available. $)% 'inding arbitration tends to be more advantageous for public&service or"ers here it is the only available means of settling labor disputes ith such or"ers. $*% Most categories of public&sector or"ers have no counterparts in the private sector. $+% A stri"e by or"ers in a local government is unli"ely to be settled ithout help from an arbitrator.

CRITICAL REASONING TEST SECTION 6 30 MINUTES 20 QUESTIONS 1. <ural households have more purchasing po er than do urban or suburban households at the same income level, since some of the income urban and suburban households use for food and shelter can be used by rural households for other needs. 8hich of the follo ing inferences is best supported by the statement made above! $A% The average rural household includes more people than does the average urban or suburban household. $'% <ural households have lo er food and housing costs than do either urban or suburban households. $)% Suburban households generally have more purchasing po er than do either rural or urban households. $*% The median income of urban and suburban households is generally higher than that of rural households. $+% All three types of households spend more of

23

GMAT

their income on food and housing than on all other purchases combined. -. 1n 1C62 state border colleges in Te/as lost the enrollment of more than half, on average, of the Me/ican nationals they had previously served each year. Teaching faculties have alleged that this e/treme drop resulted from a rise in tuition for international and out&of&state students from 503 to 51-3 per credit hour. 8hich of the follo ing, if feasible, offers the best prospects for alleviating the problem of the drop in enrollment of Me/ican nationals as the teaching faculties assessed it! $A% 4roviding grants&in&aid to Me/ican nationals to study in Me/ican universities $'% Allo ing Me/ican nationals to study in Te/as border colleges and to pay in&state tuition rates, hich are the same as the previous international rate $)% <eemphasiBing the goals and mission of the Te/as state border colleges as serving both in&state students and Me/ican nationals $*% 1ncreasing the financial resources of Te/as colleges by raising the tuition for in&state students attending state institutions $+% ,ffering career counseling for those Me/ican nationals ho graduate from state border colleges and intend to return to Me/ico

.. Affirmative action is good business. So asserted the National Association of Manufacturers hile urging retention of an e/ecutive order re#uiring some federal contractors to set numerical goals for hiring minorities and omen. G*iversity in or" force participation has produced ne ideas in management, product development, and mar"eting,H the association claimed. The associations argument as it is presented in the passage above ould be most strengthened if hich of the follo ing ere true! $A% The percentage of minority and omen or"ers in business has increased more slo ly than many minority and omens groups ould prefer. $'% Those businesses ith the highest percentages of minority and omen or"ers are those that have been the most innovative and profitable $)% *isposable income has been rising as fast among minorities and omen as among the population as a hole.

21

GMAT

$*% The biggest gro th in sales in the manufacturing sector has come in industries that mar"et the most innovative products. $+% <ecent improvements in management practices have allo ed many manufacturers to e/perience enormous gains in or"er productivity. Luestions 0&2 refer to the follo ing. 1f the airspace around centrally located airports ere restricted to commercial airliners and only those private planes e#uipped ith radar, most of the private&plane traffic ould be forced to use outlying airfields. Such a reduction in the amount of private&plane traffic ould reduce the ris" of midair collision around the centrally located airports. 0. The conclusion dra n in the first sentence depends on hich of the follo ing assumptions! $A% ,utlying airfields ould be as convenient as centrally located airports for most pilots of private planes. $'% Most outlying airfields are not e#uipped to handle commercial&airline traffic. $)% Most private planes that use centrally located airports are not e#uipped ith radar. $*% )ommercial airliners are at greater ris" of becoming involved in midair collisions than are private planes. $+% A reduction in the ris" of midair collision ould eventually lead to increases in commercial&airline traffic.

2. 8hich of the follo ing, if true, ould most strengthen the conclusion dra n in the second sentence! $A% )ommercial airliners are already re#uired by la to be e#uipped ith e/tremely sophisticated radar systems. $'% )entrally located airports are e/periencing over& cro ded airspace primarily because of sharp increases in commercial&airline traffic. $)% Many pilots of private planes ould rather buy radar e#uipment than be e/cluded from centrally located airports. $*% The number of midair collisions that occur near centrally located airports has decreased in recent years. $+% 4rivate planes not e#uipped ith radar systems cause a disproportionately large number of midair collisions around centrally located airports. ;. 8hich of the follo ing best completes the passage belo ! +stablished companies

2-

GMAT

concentrate on defending hat they already have. 7. Most archaeologists have held )onse#uently, they tend not to be innovative that people first reached the themselves and tend to underestimate the effects of the Americas less than -3,333 innovations of others. The clearest e/ample of this years ago by crossing a land defensive strategy is the fact that. bridge into North America. $A% ballpoint pens and soft&tip mar"ers have eliminated 'ut recent discoveries of the traditional mar"et for fountain pens, clearing human shelters in South the ay for the mar"eting of fountain pens as America dating from .-,333 lu/ury or prestige items years ago have led researchers $'% a highly successful automobile as introduced by to speculate that people the same company that had earlier introduced a arrived in South America first, model that had been a dismal failure after voyaging across the $)% a once&successful manufacturer of slide rules 4acific, and then spread reacted to the introduction of electronic calculators north ard. by trying to ma"e better slide rules 8hich of the follo ing, if it $*% one of the first models of modern accounting ere discovered, ould be machines, designed for use in the ban"ing pertinent evidence against the industry, as purchased by a public library as ell speculation above! as by ban"s $+% the inventor of a commonly used anesthetic did not $A% A roc" shelter near intend the product to be used by dentists, ho 4ittsburgh, 4ennsylvania, currently account for almost the entire mar"et for contains evidence of use that drug by human beings 1C,333 years ago. $'% Some North American sites of human habitation predate any sites found in South America. $)% The climate is armer at the .-,333&year&old south American site than at the oldest "no n North American site. $*% The site in South America that as occupied .-,333 years ago as continuously occupied until ;,333 years ago.
2.

GMAT

C. Since the mayors publicity campaign for Greenvilles bus service began si/ months ago, 6. 1n Asia, here palm trees are non&native, the treesN morning automobile traffic flo ers have traditionally been pollinated by hand, into the midto n area of the hich has "ept palm fruit productivity unnaturally lo . city has decreased seven 8hen eevils "no n to be efficient pollinators of palm percent. *uring the same flo ers ere introduced into Asia in 1C63, palm fruit period, there has been an productivity increasedby up to fifty percent in some e#uivalent rise in the number areasbut then decreased sharply in 1C60. of persons riding buses into the midto n area. ,bviously, 8hich of the follo ing statements, if true, ould best the mayors publicity e/plain the 1C60 decrease in productivity! campaign has convinced many $A% 4rices for palm fruit fell bet een 1C63 and 1C60 people to leave their cars at follo ing the rise in production and a concurrent home and ride the bus to fall in demand. or".. $'% 1mported trees are often more productive than 8hich of the follo ing, if native trees because the imported ones have left true, casts the most serious behind their pests and diseases in their native doubt on the conclusion dra n lands. above! $)% <apid increases in productivity tend to deplete trees of nutrients needed for the development of the fruit&producing female flo ers. $*% The eevil population in Asia remained at appro/imately the same level bet een 1C63 and 1C60. $+% 4rior to 1C63 another species of insect pollinated the Asian palm trees, but not as efficiently as the species of eevil that as introduced in 1C63. $A% 9ares for all bus routes in Greenville have risen an average of five percent during the past si/ months. $'% The mayor of Greenville rides the bus to )ity Iall in the citys midto n area. $)% <oad reconstruction has greatly reduced the number of lanes available to commuters in ma(or streets leading to the midto n area during the past si/ months. $*% The number of buses entering the midto n area

$+% The last 1ce Age, bet een 11,233 and -3,333 years ago, considerably lo ered orld ide sea levels.

20

GMAT

of Greenville during the morning hours is e/actly the same no as it as one year ago. $+% Surveys sho that longtime bus riders are no more satisfied ith the Greenville bus service than they ere before the mayors publicity campaign began. 13. 1n the aftermath of a orld ide stoc"&mar"et crash, )ountry T claimed that the severity of the stoc"& mar"et crash it e/perienced resulted from the accelerated process of denationaliBation many of its industries under ent shortly before the crash. 8hich of the follo ing, if it could be carried out, ould be most useful in an evaluation of )ountry Ts assessment of the causes of the severity of its stoc"& mar"et crash! $A% )alculating the average loss e/perienced by individual traders in )ountry T during the crash $'% :sing economic theory to predict the most li"ely date of the ne/t crash in )ountry T $)% )omparing the total number of shares sold during the orst days of the crash in )ountry T to the total number of shares sold in )ountry T (ust prior to the crash. $*% )omparing the severity of the crash in )ountry T to the severity of the crash in countries other ise economically similar to )ountry T that have not e/perienced recent denationaliBation $+% )omparing the long&term effects of the crash on the purchasing po er of the currency of )ountry T to the immediate, more severe short&term effects of the crash on the purchasing po er of the currency of )ountry T

11. 8ith the emergence of biotechnology companies, it as feared that they ould impose silence about proprietary results on their in&house researchers and their academic consultants. This constraint, in turn, ould slo the development of biological science and engineering. 8hich of the follo ing, if true, ould tend to ea"en most seriously the prediction of scientific secrecy described above! $A% 'iotechnological research funded by industry has reached some conclusions that are of ma(or scientific importance. $'% 8hen the results of scientific research are "ept secret, independent researchers are unable to build on those results. $)% Since the research priorities of biotechnology companies are not the same as those of academic institutions, the financial support of research by such companies distorts the research agenda. $*% To enhance the companiesN standing in the scientific

22

GMAT

community, the biotechnology companies encourage employees to publish their results, especially results that are important. $+%'iotechnology companies devote some of their research resources to problems that are of fundamental scientific importance and that are not e/pected to produce immediate practical applications. 1-. Some people have #uestioned the (udges ob(ectivity in cases of se/ discrimination against omen. 'ut the record sho s that in si/ty percent of such cases, the (udge has decided in favor of the omen. This record demonstrates that the (udge has not discriminated against omen in cases of se/ discrimination against omen. The argument above is fla ed in that it ignores the possibility that $A% a large number of the (udges cases arose out of allegations of se/ discrimination against omen $'% many (udges find it difficult to be ob(ective in cases of se/ discrimination against omen $)% the (udge is biased against omen defendants or plaintiffs in cases that do not involve se/ discrimination $*% the ma(ority of the cases of se/ discrimination against omen that have reached the (udges court have been appealed from a lo er court $+% the evidence sho s that the omen should have on in more than si/ty percent of the (udges cases involving se/ discrimination against omen

1.. The tobacco industry is still profitable and pro(ections are that it ill remain so. 1n the :nited States this year, the total amount of tobacco sold by tobacco&farmers has increased, even though the number of adults ho smo"e has decreased. +ach of the follo ing, if true, could e/plain the simultaneous increase in tobacco sales and decrease in the number of adults ho smo"e +>)+4T. $A% *uring this year, the number of omen ho have begun to smo"e is greater than the number of men ho have #uit smo"ing. $'% The number of teen&age children ho have begun to smo"e this year is greater than the number of adults ho have #uit smo"ing during the same period. $)% *uring this year, the number of nonsmo"ers ho have begun to use che ing tobacco or snuff is greater than the number of people ho have #uit smo"ing. $*% The people ho have continued to smo"e

2;

GMAT

consume more tobacco per person than they did in 12. ,n the basis of a decrease in the past. the college&age population, $+% More of the cigarettes made in the :nited States many colleges no this year ere e/ported to other countries than as anticipate increasingly the case last year. smaller freshman classes 10. Dale has more nutritional value than spinach. 'ut each year. Surprised by a 03 since collard greens have more nutritional value than percent increase in #ualified lettuce, it follo s that "ale has more nutritional value applicants over the previous than lettuce. year, ho ever, administrators at Nice Any of the follo ing, if introduced into the argument )ollege no plan to hire as an additional premise, ma"es the argument above more faculty for courses logically correct +>)+4T= ta"en by all freshmen. $A% )ollard greens have more nutritional value than 8hich of the follo ing "ale. statements about Nice $'% Spinach has more nutritional value than lettuce. )olleges current #ualified $)% Spinach has more nutritional value than collard applicants, if true, ould greens. strongly suggest that the $*% Spinach and collard greens have the same administrators plan is nutritional value. fla ed! $+% Dale and collard greens have the same nutritional value. $A% A substantially higher percentage than usual plan to study for advanced degrees after graduation from college. $'% According to their applications, their level of participation in e/tracurricular activities and varsity sports is unusually high. $)% According to their applications, none of them lives in a foreign country. $*% A substantially lo er percentage than usual rate
27

GMAT

Nice )ollege as their first choice among the colleges to hich they are applying. $+% A substantially lo er percentage than usual list mathematics as their intended ma(or. Luestions 1;&17 are based on the follo ing. A researcher discovered that people ho have lo levels of immune&system activity tend to score much lo er on tests of mental health than do people ith normal or high immune&system activity. The researcher concluded from this e/periment that the immune system protects against mental illness as ell as against physical disease. 1;. The researchers conclusion depends on hich of the follo ing assumptions! $A% Iigh immune&system activity protects against mental illness better than normal immune&system activity does. $'% Mental illness is similar to physical disease in its effects on body system. $)% 4eople ith high immune&system activity cannot develop mental illness. $*% Mental illness does not cause peoples immune& system activity to decrease. $+% 4sychological treatment of mental illness is not as effective as is medical treatment.

17. The researchers conclusion ould be most seriously ea"ened if it ere true that $A% there as a one&year delay bet een the completion of a pilot study for the e/periment and the initiation of the e/periment itself $'% peoples levels of immune&system activity are not affected by their use of medications $)% a fe people ith high immune&system activity had scores on the test of mental health that ere similar to the scores of people ho had normal immune&system activity $*% people ho have lo immune&system activity tend to contract more viral infections than do people ith normal or high immune&system activity $+% high levels of stress first cause mental illness and then cause decreased immune&system activity in normal individuals 16. The value of a product is determined by the ratio of its #uality to its price. The higher the value of a product, the better ill be

26

GMAT

its competitive position. Therefore, either increasing the #uality or lo ering the price of a given product ill increase the li"elihood that consumer ill select that product rather than a competing one.

1C. 1n ?anuary there as a large drop in the number of ne houses sold, because interest rates for mortgages 8hich of the follo ing, if true, ould most ere falling and many strengthen the conclusion dra n above! consumers ere aiting to $A% 1t is possible to increase both the #uality and the see ho lo the rates ould price of a product ithout changing its competitive go. This large sales drop position. as accompanied by a sharp $'% 9or certain segments of the population of rise in the average price of consumers, higher&priced brands of some product ne houses sold. lines are preferred to the lo er&priced brands. 8hich of the follo ing, if $)% )ompeting products often try to appeal to true, best e/plains the sharp different segments of the population of consumers. rise in the average price of $*% The competitive position of a product can be ne houses! affected by such factors as advertising and brand $A% Sales of higher&priced loyalty. houses ere unaffected $+% )onsumers perceptions of the #uality of a by the sales drop because product are based on the actual #uality of the their purchasers have product. fe er constraints limiting the total amount they pay. $'% @abor agreements of builders ith construction unions are not due to e/pire until the ne/t ?anuary. $)% The prices of ne houses have been rising slo ly over the past three years because there is an increasing shortage of housing. $*% There as a greater amount of moderate& priced housing available for resale by o ners during ?anuary than in the
2C

GMAT

preceding three months. $+% 1nterest rates for home mortgages are e/pected to rise sharply later in the year if predictions of increased business activity in general prove to be accurate.

CRITICAL REASONING TEST SECTION 7 30 MINUTES 20 QUESTIONS

1. A milepost on the to path -3. Seven countries signed a treaty binding each of them read G-1H on the side facing to perform specified actions on a certain fi/ed date, the hi"er as she approached it ith the actions of each conditional on simultaneous and G-.H on its bac". She action ta"en by the other countries. +ach country as reasoned that the ne/t milepost also to notify the si/ other countries hen it had for ard on the path ould completed its action. indicate that she as half ay bet een one end of the path The simultaneous&action provision of the treaty and the other. Io ever, the leaves open the possibility that milepost one mile further on $A% the compliance date as sub(ect to read G-3H facing her and G-0H postponement, according to the terms of the treaty behind. $'% one of the countries might not be re#uired to ma"e any changes or ta"e any steps in order to 8hich of the follo ing, if comply ith the treaty, hereas all the other true, ould e/plain the countries are so re#uired. discrepancy described above! $)% each country might have a ell&founded e/cuse, $A% The numbers on the ne/t based on the provision, for its o n lac" of milepost had been compliance reversed. $*% the treaty specified that the signal for one of the $'% The numbers on the countries to initiate action as notification by the mileposts indicate other countries that they had completed action "ilometers, not miles. $+% there as ambiguity ith respect to the date after $)% The facing numbers hich all actions contemplated in the treaty are to indicate miles to the end be complete. of the path, not miles from the beginning. $*% A milepost as missing bet een the t o the hi"er encountered. $+% The mileposts had originally been put in place for the use of mountain bi"ers, not for
;3

GMAT

hi"ers. .. Guitar strings often go -. Airline= Ne ly developed collision&avoidance systems, GdeadHbecome less although not fully tested to discover potential responsive and bright in tone malfunctions, must be installed immediately in after a fe ee"s of intense passenger planes. Their mechanical arnings enable use. A researcher hose son is pilots to avoid crashes. a classical guitarist 4ilots= 4ilots ill not fly in planes ith collision& hypothesiBed that dirt and oil, avoidance systems that are not fully tested. rather than changes in the Malfunctioning systems could mislead pilots, causing material properties of the crashes. string, ere responsible. The pilots ob(ection is most strengthened if hich of 8hich of the follo ing the follo ing is true! investigations is most li"ely to yield significant information $A% 1t is al ays possible for mechanical devices to that ould help to evaluate the malfunction. researchers hypothesis! $'% ?et engines, although not fully tested hen first put into use, have achieved e/emplary performance $A% *etermining if a metal and safety records. alloy is used to ma"e the $)% Although collision&avoidance systems ill enable strings used by classical pilots to avoid some crashes, the li"ely guitarists malfunctions of the not&fully&tested systems ill $'% *etermining hether cause even more crashes. classical guitarists ma"e $*% Many airline collisions are caused in part by the their strings go dead e/haustion of over or"ed pilots. faster than do fol" $+% )ollision&avoidance systems, at this stage of guitarists development, appear to have or"ed better in $)% *etermining hether passenger planes than in cargo planes during identical lengths of string, e/perimental flights made over a si/&month of the same gauge, go period. dead at different rates hen strung on various brands of guitars. $*% *etermining hether a dead string and a ne string produce different #ualities of sound $+% *etermining hether smearing various substances on ne guitar
;1

GMAT

2. T o decades after the +merald <iver *am as built, none of 0. Most consumers do not get much use out of the sports the eight fish species native to e#uipment they purchase. 9or e/ample, seventeen the +merald <iver as still percent of the adults in the :nited States o n (ogging reproducing ade#uately in the shoes, but only forty&five percent of the o ners (og river belo the dam. Since the more than once a year, and only seventeen percent (og dam reduced the annual range more than once a ee". of ater temperature in the 8hich of the follo ing, if true, casts most doubt on the river belo the dam from 23 claim that most consumers get little use out of the degrees to ; degrees, scientists sports e#uipment they purchase! have hypothesiBed that sharply rising ater temperatures must $A% ?oggers are most susceptible to sports in(uries be involved in signaling the during the first si/ months in hich they (og. native species to begin the $'% ?oggers often e/aggerate the fre#uency ith hich reproductive cycle. they (og in surveys designed to elicit such information. 8hich of the follo ing $)% Many consumers purchase (ogging shoes for use in statements, if true, ould most activities other than (ogging. strengthen the scientists $*% )onsumers ho ta"e up (ogging often purchase an hypothesis! athletic shoe that can be used in other sports. $+% ?oggers ho (og more than once a ee" are often $A% The native fish species ere active participants in other sports as ell. still able to reproduce only in side streams of the river belo the dam here the annual temperature range remains appro/imately 23 degrees. $'% 'efore the dam as built, the +merald <iver annually overflo ed its ban"s, creating bac" aters that ere critical breeding areas for the native species of fish. $)% The lo est recorded temperature of the +merald <iver before the dam as built as .0
;-

strings causes them to go dead

GMAT

degrees, hereas the lo est recorded temperature of the river after the dam as built has been 0. degrees. $*%Nonnative species of fish, introduced into the +merald <iver after the dam as built, have begun competing ith the declining native fish species for food and space. $+% 9ive of the fish species native to the +merald <iver are not native to any other river in North America. ;. 1t is true that it is against international la to sell plutonium to countries that do not yet have nuclear eapons. 'ut if :nited States companies do not do so, companies in other countries ill. 8hich of the follo ing is most li"e the argument above in its logical structure!

7. 1n recent years many cabinetma"ers have been inning acclaim as artists. 'ut since furniture must be useful, cabinetma"ers must e/ercise their craft ith an eye to the practical utility of their product. 9or this reason, cabinetma"ing is not art. 8hich of the follo ing is an assumption that supports dra ing the conclusion above from the reason given for that conclusion! $A% Some furniture is made to be placed in museums, here it ill not be used by anyone. $'% Some cabinetma"ers are more concerned than others ith the practical utility of the products they produce. $)% )abinetma"ers should be more concerned ith the practical utility of their products than they currently are. $*% An ob(ect is not an art ob(ect if its ma"er pays attention to the ob(ects practical utility. $+% Artists are not concerned ith the monetary value of their products.

$A% 1t is true that it is against the police departments policy to negotiate ith "idnappers. 'ut if the police ant to prevent loss of life, they must negotiate in some cases. $'% it is true that it is illegal to refuse to register for military service. 'ut there is a long tradition in the :nited States of conscientious ob(ection to serving in the armed forces. $)% 1t is true that it is illegal for a government official to participate in a transaction in hich there is an apparent conflict of interest. 'ut if the facts are e/amined carefully, it ill clearly be seen that there as no actual conflict of interest in the defendants case. $*% 1t is true that it is against the la to burglariBe peoples homes. 'ut someone else certainly ould have burglariBed that house if the defendant had not done so first. $+% 1t is true that company policy forbids supervisors to fire employees ithout t o ritten arnings. 'ut there have been many supervisors ho have 6. Although custom prosthetic disobeyed this policy. bone replacements produced
;.

GMAT

C. +/tinction is a process that can depend on a variety of ecological, geographical, and physiological variables. These variables affect different species of organisms in different ays, and should, therefore, yield a random $A% The amount of time a patient spends in surgery pattern of e/tinctions. versus the amount of time spent recovering from Io ever, the fossil record surgery sho s that e/tinction occurs in $'% The amount by hich the cost of producing custom a surprisingly definite pattern, replacements has declined ith the introduction of ith many species vanishing the ne techni#ue for producing them at the same time. $)%The degree to hich the use of custom replacements 8hich of the follo ing, if is li"ely to reduce the need for repeat surgery true, forms the best basis for at hen compared ith the use of ordinary least a partial e/planation of replacements the patterned e/tinctions $*% The degree to hich custom replacements revealed by the fossil record! produced ith the ne techni#ue are more $A% Ma(or episodes of carefully manufactured than are ordinary e/tinction can result from replacements idespread $+% The amount by hich custom replacements environmental produced ith the ne techni#ue ill drop in cost disturbances that affect as the production procedures become standardiBed numerous different and applicable on a larger scale species. $'% )ertain e/tinction episodes selectively affect organisms ith particular sets of characteristics uni#ue to their species. $)% Some species become e/tinct because of accumulated gradual changes in their local environments. $*% 1n geologically recent
;0

through a ne computer&aided design process ill cost more than t ice as much as ordinary replacements, custom replacements should still be cost&effective. Not only ill surgery and recovery time be reduced, but custom replacements should last longer, thereby reducing the need for further hospital stays. 8hich of the follo ing must be studied in order to evaluate the argument presented above!

GMAT

times, for hich there is no fossil record, human 11.)ertain messenger molecules intervention has changed the pattern of e/tinctions. fight damage to the lungs $+% Species that are idely dispersed are the least li"ely from no/ious air by telling to become e/tinct. the muscle cells encircling the lungs air ays to 13. Neither a rising standard of living nor balanced trade, contract. This partially seals by itself, establishes a countrys ability to compete in off the lungs. An asthma the international mar"etplace. 'oth are re#uired attac" occurs hen the simultaneously since standards of living can rise messenger molecules are because of gro ing trade deficits and trade can be activated unnecessarily, in balanced by means of a decline in a countrys response to harmless things standard of living. li"e pollen or household dust. 1f the facts stated in the passage above are true, a 8hich of the follo ing, if proper test of a countrys ability to be competitive is true, points to the most its ability to serious fla of a plan to develop a medication that $A% balance its trade hile its standard of living rises ould prevent asthma $'% balance its trade hile its standard of living falls attac"s by bloc"ing receipt $)% increase trade deficits hile its standard of living of any messages sent by the rises messenger molecules $*% decrease trade deficits hile its standard of living referred to above! falls $+% "eep its standard of living constant hile trade deficits rise. $A% <esearchers do not yet "no ho the body produces the messenger molecules that trigger asthma attac"s. $'% <esearchers do not yet "no hat ma"es one persons messenger molecules more easily activated than anothers. $)% Such a medication ould not become available for several years, because of long lead times in both development and

;2

GMAT

manufacture. $*% Such a medication ould be unable to distinguish bet een messages triggered by pollen and household dust and messages triggered by no/ious air. $+% Such a medication ould be a preventative only and ould be unable to alleviate an asthma attac" once it had started. 1-. Since the routine use of antibiotics can give rise to resistant bacteria capable of surviving antibiotic environments, the presence of resistant bacteria in people could be due to the human use of prescription antibiotics. Some scientists, ho ever, believe that most resistant bacteria in people derive from human consumption of bacterially infected meat. 8hich of the follo ing statements, if true, ould most significantly strengthen the hypothesis of the scientists! $A% Antibiotics are routinely included in livestoc" feed so that livestoc" producers can increase the rate of gro th of their animals. $'% Most people ho develop food poisoning from bacterially infected meat are treated ith prescription antibiotics. $)% The incidence of resistant bacteria in people has tended to be much higher in urban areas than in rural areas here meat is of comparable #uality. $*% 4eople ho have never ta"en prescription antibiotics are those least li"ely to develop resistant bacteria. $+% @ivestoc" producers claim that resistant bacteria in animals cannot be transmitted to people through infected meat.

1.. The recent decline in the value of the dollar as triggered by a prediction of slo er economic gro th in the coming year. 'ut that prediction ould not have adversely affected the dollar had it not been for the governments huge budget deficit, hich must therefore be decreased to prevent future currency declines. 8hich of the follo ing, if true, ould most seriously ea"en the conclusion about ho to prevent future currency declines! $A% The government has made little attempt to reduce the budget deficit. $'% The budget deficit has not caused a slo do n in economic gro th. $)% The value of the dollar declined several times in the year prior to the recent prediction of slo er economic gro th. $*% 'efore there as a large budget deficit, predictions of slo er economic gro th fre#uently caused declines in the dollars value. $+% 8hen there is a large budget deficit, other

;;

GMAT

events in addition to predictions of slo er economic gro th sometimes trigger declines in currency value. 10. 8hich of the follo ing best completes the passage belo ! At a recent conference on environmental threats to the North Sea, most participating countries favored uniform controls on the #uality of effluents, hether or not specific environmental damage could be attributed to a particular source of effluent. 8hat must, of course, be sho n, in order to avoid e/cessively restrictive controls, is that AAAAAAAAAAA. $A% any uniform controls that are adopted are li"ely to be implemented ithout delay $'% any substance to be made sub(ect to controls can actually cause environmental damage $)% the countries favoring uniform controls are those generating the largest #uantities of effluents $*% all of any given pollutant that is to be controlled actually reaches the North Sea at present $+% environmental damage already inflicted on the North Sea is reversible

12. Traditionally, decision& ma"ing by managers that is reasoned step&by&step has been considered preferable to intuitive decision& ma"ing. Io ever, a recent study found that top managers used intuition significantly more than did most middle&or lo er&level managers. This confirms the alternative vie that intuition is actually more effective than careful, methodical reasoning. The conclusion above is based on hich of the follo ing assumptions! $A% Methodical, step&by& step reasoning is inappropriate for ma"ing many real&life management decisions. $'% Top managers have the ability to use either intuitive reasoning or methodical, step&by&step reasoning in ma"ing decisions. $)% The decisions made by middle&and lo er&level managers can be made as easily by using methodical reasoning as by using intuitive reasoning. $*% Top managers use

;7

GMAT

intuitive reasoning in ma"ing the ma(ority of their 17. )orrectly measuring the decisions. productivity of service $+% Top managers are more effective at decision& or"ers is comple/. ma"ing than middle&or lo er&level managers )onsider, for e/ample, 1;. The imposition of #uotas limiting imported steel ill postal or"ers= they are not help the big American steel mills. 1n fact, the often said to be more #uotas ill help Gmini&millsH flourish in the :nited productive if more letters States. Those small domestic mills ill ta"e more are delivered per postal business from the big Americal steel mills than or"er. 'ut is this really ould have been ta"en by the foreign steel mills in true! hat if more letters the absence of #uotas. are lost or delayed per or"er at the same time that 8hich of the follo ing, if true, ould cast the most more are delivered! serious doubt on the claim made in the last sentence The ob(ection implied above above! to the productivity measure described is based on doubts $A% Luality rather than price is a ma(or factor in about the truth of hich of determining the type of steel to be used for a the follo ing statements! particular application. $'% 9oreign steel mills have long produced grades of $A% 4ostal or"ers are steel comparable in #uality to the steel produced representative of service by the big American mills. or"ers in general. $)% American #uotas on imported goods have often $'% The delivery of letters is induced other countries to impose similar #uotas the primary activity of the on American goods. postal service. $*% *omestic Gmini&millsH consistently produce $)% 4roductivity should be better grades of steel than do the big American ascribed to categories of mills. or"ers, not to $+% *omestic Gmini&millsH produce lo &volume, individuals. specialiBed types of steels that are not produced by $*% The #uality of services the big American steel mills. rendered can appropriately be ignored in computing productivity. $+% The number of letters delivered is relevant to measuring the productivity of postal
;6

GMAT

or"ers. 16. Male bo erbirds construct elaborately decorated nests, or bo ers. 'asing their (udgment on the fact that different local populations of bo erbirds of the same species build bo ers that e/hibit different building and decorative styles, researchers have concluded that the bo erbirds building styles are a culturally ac#uired, rather than a genetically transmitted, trait. 8hich of the follo ing, if true, ould most strengthen the conclusion dra n by the researchers! $A% There are more common characteristics than there are differences among the bo er&building styles of the local bo erbird population that has been studied most e/tensively $'% Joung male bo erbirds are inept at bo er& building and apparently spend years atching their elders before becoming accomplished in the local bo er style. $)% The bo ers of one species of bo erbird lac" the to ers and ornamentation characteristic of the bo ers of most other species of bo erbird. $*% 'o erbirds are found only in Ne Guinea and Australia, here local populations of the birds apparently seldom have contact ith one another. $+% 1t is ell "no n that the song dialects of some songbirds are learned rather than transmitted genetically. 1C. A greater number of ne spapers are sold in To n S than in To n T. Therefore, the citiBens of To n S are better informed about ma(or orld events than are the citiBens of To n T. +ach of the follo ing, if true, ea"ens the conclusion above +>)+4T= $A% To n S has a larger population than To n T. $'% Most citiBens of To n T or" in To n S and buy their ne spapers there. $)% The average citiBen of To n S spends less time reading ne spapers than does the average citiBen of To n T. $*% A ee"ly ne spaper restricted to the coverage of local events is published in To n S. $+% The average ne sstand price of ne spapers sold in To n S in lo er than the average price of ne spapers sold in To n T. -3. ,ne analyst predicts that Iong Dong can retain its capitalist ays after it becomes part of mainland )hina in 1CC7 as long as a capitalist Iong Dong is
;C

GMAT

useful to )hinaE that a capitalist Iong Dong ill be CRITICAL REASONING useful to )hina as long as Iong Dong is prosperousE TEST SECTION 8 and that Iong Dong ill remain prosperous as long 30 MINUTES 20 as it retains its capitalist ays. QUESTIONS 1f the predictions above are correct, hich of the follo ing further predictions can logically be derived 1. A drug that is highly effective in treating many types of from them! infection can, at present, be $A% 1f Iong Dong fails to stay prosperous, it ill no obtained only from the bar" of longer remain part of mainland )hina. the ibora, a tree that is #uite $'% 1f Iong Dong retains its capitalist ays until rare in the ild. 1t ta"es the 1CC7, it ill be allo ed to do so after ard. bar" of 2,333 tree to ma"e one $)% 1f there is a orld economic crisis after 1CC7, it "ilogram of the drug. 1t ill not adversely affect the economy of Iong follo s, therefore, that Dong. continued production of the $*% Iong Dong ill be prosperous after 1CC7 drug must inevitably lead to $+% The citiBens of Iong Dong ill have no the iboras e/tinction. restrictions placed on them by the government of mainland )hina. 8hich of the follo ing, if true, most seriously ea"ens the argument above! $A% The drug made from ibora bar" is dispensed to doctors from a central authority. $'% The drug made from ibora bar" is e/pensive to produce. $)% The leaves of the ibora are used in a number of medical products. $*% The ibora can be propagated from cuttings and gro n under cultivation. $+% The ibora generally gro s in largely inaccessible places.
73

GMAT

-. Iigh levels of fertiliBer and pesticides, needed hen farmers try to produce high yield of the same crop year after year, pollute ater supplies. +/perts therefore urge farmers to diversify their crops and to rotate their plantings yearly.

.. Shelby 1ndustries manufactures and sells the same gauges as ?ones 1ndustries. +mployee ages account for forty percent of the cost of manufacturing To receive governmental price&support benefits for a gauges at both Shelby crop, farmers must have produced that same crop for 1ndustries and ?ones the past several years. 1ndustries. Shelby 1ndustries is see"ing a competitive The statements above, if true, best support hich of the advantage over ?ones follo ing conclusions! 1ndustries. Therefore, to $A% The rules for governmental support of farm prices promote this end, Shelby or" against efforts to reduce ater pollution. 1ndustries should lo er $'% The only solution to the problem of ater pollution employee ages. from fertiliBers and pesticides is to ta"e farmland out of production. 8hich of the follo ing, if $)% 9armers can continue to ma"e a profit by rotating true, ould most ea"en the diverse crops, thus reducing costs for chemicals, argument above! but not by planting the same crop each year. $A% 'ecause they ma"e a $*% Ne farming techni#ues ill be developed to ma"e small number of precision it possible for farmers to reduce the application of instruments, gauge fertiliBers and pesticides. manufacturers cannot $+% Governmental price supports for farm products are receive volume discounts set at levels that are not high enough to allo on ra materials. farmers to get out of debt. $'% @o ering ages ould reduce the #uality of employee or", and this reduced #uality ould lead to lo ered sales. $)% ?ones 1ndustries has ta"en a ay t enty percent of Shelby 1ndustries business over the last year. $*% Shelby 1ndustries pays its employees, on average,
71

GMAT

ten percent more than does ?ones 1ndustries. 2. @arge national budget deficits $+% Many people ho or" for manufacturing plants do not cause large trade live in areas in hich the manufacturing plant they deficits. 1f they did, countries or" for is the only industry. ith the largest budget deficits 0. Some communities in 9lorida are populated almost ould also have the largest e/clusively by retired people and contain fe , if any, trade deficits. 1n fact, hen families ith small children. Jet these communities are deficit figures are ad(usted so home to thriving businesses specialiBing in the rental of that different countries are furniture for infants and small children. reliably comparable to each other, there is no such 8hich of the follo ing, if true, best reconciles the correlation. seeming discrepancy described above! 1f the statements above are all $A% The businesses specialiBing in the rental of true, hich of the follo ing childrens furniture buy their furniture from can properly be inferred on the distributors outside of 9lorida. basis of them! $'% The fe children ho do reside in these communities all "no each other and often ma"e $A% )ountries ith large overnight visits to one anothers houses. national budget deficits $)% Many residents of these communities ho move tend to restrict foreign fre#uently prefer renting their furniture to buying trade. it outright. $'% <eliable comparisons of $*% Many residents of these communities must provide the deficit figures of one for the needs of visiting grandchildren several country ith those of ee"s a year. another are impossible. $+% )hildrens furniture available for rental is of the $)% <educing a countrys same #uality as that available for sale in the stores. national budget deficit ill not necessarily result in a lo ering of any trade deficit that country may have. $*% 8hen countries are ordered from largest to smallest in terms of population, the smallest countries generally have the smallest budget and trade deficits.
7-

GMAT

$+% )ountries ith the largest trade deficits never have similarly large national budget deficits. ;. G9ast cycle timeH is a strategy of designing a manu& facturing organiBation to eliminate bottlenec"s and delays in production. Not only does it speed up production, but it also assures #uality. The reason is that the bottlenec"s and delays cannot be eliminated unless all or" is done right the first time. The claim about #uality made above rests on a #ues& tionable presupposition that $A% any fla in or" on a product ould cause a bottlenec" or delay and so ould be prevented from occurring on a Gfast cycleH production line $'% the strategy of Gfast cycle timeH ould re#uire fundamental rethin"ing of product design $)% the primary goal of the organiBation is to produce a product of une/celled #uality, rather than to generate profits for stoc"holders $*% Gfast cycle timeH could be achieved by shaving time off each of the component processes in production cycle $+% Gfast cycle timeH is a concept in business strategy that has not yet been put into practice in a factory

7. Many brea"fast cereals are fortified ith vitamin supplements. Some of these cereals provide 133 percent of the recommended daily re#uirement of vitamins. Nevertheless, a ell&balanced brea"fast, including a variety of foods, is a better source of those vitamins than are such fortified brea"fast cereals alone. 8hich of the follo ing, if true, ould most strongly support the position above! $A% 1n many foods, the natural combination of vitamins ith other nutrients ma"es those vitamins more usable by the body than are vitamins added in vitamin supplements. $'% 4eople ho regularly eat cereals fortified ith vitamin supplements sometimes neglect to eat the foods in hich the vitamins occur naturally. $)%9oods often must be fortified ith vitamin supplements because naturally occurring vitamins are removed during processing. $*% :nprocessed cereals are naturally high in several

7.

GMAT

of the vitamins that are usually added to fortified C. A famous singer recently on brea"fast cereals. a la suit against an $+% )ereals containing vitamin supplements are no advertising firm for using harder to digest than similar cereals ithout added another singer in a vitamins. commercial to evo"e the 6. 8hich of the follo ing best completes the passage famous singers ell&"no n belo ! rendition of a certain song. The more orried investors are about losing their As a result of the la suit, money, the more they ill demand a high potential advertising firms ill stop return on their investmentE great ris"s must be offset by using imitators in the chance of great re ards. This principle is the commercials. Therefore, fundamental one in determining interest rates, and it is advertising costs ill rise, illustrated by the fact that . since famous singers services cost more than $A% successful investors are distinguished by an ability those of their imitators. to ma"e very ris"y investments ithout orrying The conclusion above is about their money based on hich of the $'% lenders receive higher interest rates on unsecured follo ing assumptions! loans than on loans bac"ed by collateral $)% in times of high inflation, the interest paid to $A% Most people are unable depositors by ban"s can actually be belo the rate to distinguish a famous of inflation singers rendition of a $*% at any one time, a commercial ban" ill have a song from a good single rate of interest that it ill e/pect all of its imitators rendition of the individual borro ers to pay same song. $+% the potential return on investment in a ne $'% )ommercials using company is typically lo er than the potential famous singers are return on investment in a ell&established usually more effective company than commercials using imitators of famous singers. $)% The original versions of some ell&"no n songs are unavailable for use in commercials. $*% Advertising firms ill continue to use imitators to mimic the physical
70

GMAT

mannerisms of famous singers. $+% The advertising industry ill use ell&"no n renditions of songs in commercials. 13. A certain mayor has proposed a fee of five dollars per day on private vehicles entering the city, claiming that the fee ill alleviate the citys traffic congestion. The mayor reasons that, since the fee ill e/ceed the cost of round&trip bus fare from many nearby points, many people ill s itch from using their cars to using the bus. 8hich of the follo ing statements, if true, provides the best evidence that the mayors reasoning is fla ed! $A% 4ro(ected increases in the price of gasoline ill increase the cost of ta"ing a private vehicle into the city. $'% The cost of par"ing fees already ma"es it considerably more e/pensive for most people to ta"e a private vehicle into the city than to ta"e a bus. $)% Most of the people currently riding the bus do not o n private vehicles. $*% Many commuters opposing the mayors plan have indicated that they ould rather endure traffic congestion than pay a five&dollar&per day fee. $+% *uring the average or"day, private vehicles o ned and operated by people living ithin the city account for t enty percent of the citys traffic congestion.

11. A group of children of various ages as read stories in hich people caused harm, some of those people doing so intentionally, and some accidentally. 8hen as"ed about appropriate punishments for those ho had caused harm, the younger children, unli"e the older ones, assigned punishments that did not vary according to hether the harm as done intentionally or accidentally. Jounger children, then, do not regard peoples intentions as relevant to punishment. 8hich of the follo ing, if true, ould most seriously ea"en the conclusion above! $A% 1n interpreting these stories, the listeners had to dra on a relatively mature sense of human psychology in order to tell hether harm as produced intentionally or accidentally. $'% 1n these stories, the severity of the harm produced as clearly stated.

72

GMAT

$)% Jounger children are as li"ely to produce harm unintentionally as are older children. $*% The older children assigned punishment in a ay that closely resembled the ay adults had assigned punishment in a similar e/periment. $+% The younger children assigned punishments that varied according to the severity of the harm done by the agents in the stories.

Luestions 1.&10 are based on the follo ing. The program to control the entry of illegal drugs into the country as a failure in 1C67. 1f the program had been successful, the holesale price of most illegal drugs ould not have dropped substantially in 1C67.

1-. 8hen hypnotiBed sub(ects are told that they are deaf and are then as"ed hether they can hear the hypnotist, they reply, GNo.H Some theorists try to 1.. The argument in the passage e/plain this result by arguing that the selves of depends on hich of the hypnotiBed sub(ects are dissociated into separate follo ing assumptions! parts, and that the part that is deaf is dissociated from $A% The supply of illegal the part that replies. drugs dropped substantially in 1C67. 8hich of the follo ing challenges indicates the most $'% The price paid for most serious ea"ness in the attempted e/planation illegal drugs by the described above! average consumer did not $A% 8hy does the part that replies not ans er, drop substantially in GJesH! 1C67. $'% 8hy are the observed facts in need of any special $)% *omestic production of e/planation! illegal drugs increased at $)% 8hy do the sub(ects appear to accept the a higher rate than did the hypnotists suggestion that they are deaf! entry of such drugs into $*% 8hy do hypnotiBed sub(ects all respond the same the country. ay in the situation described! $*% The holesale price of a $+% 8hy are the separate parts of the self the same fe illegal drugs for all sub(ects! increased substantially in 1C67. $+% A drop in demand for most illegal drugs in 1C67 as not the sole cause of the drop in their holesale price. 10. The argument in the passage
7;

GMAT

ould be most seriously ea"ened if it ere true that $A% in 1C67 smugglers of illegal drugs, as a group, had significantly more funds at their disposal than did the countrys customs agents $'% domestic production of illegal drugs increased substantially in 1C67 $)% the authors statements ere made in order to embarrass the officials responsible for the drug& control program $*% in 1C67 illegal drugs entered the country by a different set of routes than they did in 1C6; $+% the countrys citiBens spent substantially more money on illegal drugs in 1C67 than they did in 1C6;. 12. +/cavation of the ancient city of Dourion on the island of )yprus revealed a pattern of debris and collapsed buildings typical of to ns devastated by earth#ua"es. Archaeologists have hypothesiBed that the destruction as due to a ma(or earth#ua"e "no n to have occurred near the island in A.*..;2. 8hich of the follo ing, if true, most strongly supports the archaeologists hypothesis! $A% 'ronBe ceremonial drin"ing vessels that are often found in graves dating from years preceding and follo ing A.*..;2 ere also found in several graves near Dourion. $'% No coins minted after A.*..;2 ere found in Dourion, but coins minted before that year ere found in abundance. $)% Most modern histories of )yprus mention that an earth#ua"e occurred near the island in A.*..;2. $*% Several small statues carved in styles current in )yprus in the century
77

GMAT

bet een A.*..33 and 033 ere found in Dourion. $+% Stone inscriptions in a form of the Gree" alphabet that as definitely used in )yprus after A.*..;2 ere found in Dourion. 1;. Sales of telephones have increased dramatically over the last year. 1n order to ta"e advantage of this increase, Mammoth 1ndustries plans to e/pand production of its o n model of telephone, hile continuing its already very e/tensive advertising of this product. 8hich of the follo ing, if true, provides most support for the vie that Mammoth 1ndustries cannot increase its sales of telephones by adopting the plan outlined above! $A% Although it sells all of the telephones that it produces, Mammoth 1ndustries share of all telephone sales has declined over the last year. $'% Mammoth 1ndustries average inventory of telephones a aiting shipment to retailers has declined slightly over the last year. $)% Advertising has made the brand name of Mammoth 1ndustries telephones idely "no n, but fe consumers "no that Mammoth 1ndustries o ns this brand. $*% Mammoth 1ndustries telephone is one of three brands of telephone that have together accounted for the bul" of the last years increase in sales. $+% *espite a slight decline in the retail price, sales of Mammoth 1ndustries telephones have fallen in the last year.

17. Many institutions of higher education suffer declining enrollments during periods of economic slo do n. At t o&year community colleges, ho ever, enrollment figures boom during these periods hen many people have less money and there is more competition for (obs. +ach of the follo ing, if true, helps to e/plain the enrollment increases in t o& year community colleges described above +>)+4T= $A% *uring periods of economic slo do n, t o&year community colleges are more li"ely than four&year colleges to prepare their students for the (obs that are still available. $'% *uring periods of economic prosperity, graduates of t o&year community colleges often continue their studies at four&year colleges. $)% Tuition at most t o&year community colleges is a fraction of that at four& year colleges. $*% T o&year community colleges devote more resources than do other

76

GMAT

colleges to attracting those students especially 1C. 8hich of the follo ing, if affected by economic slo do ns. true and "no n by the $+% Students at t o&year community colleges, but not ranchers, ould best help those at most four&year colleges, can control the e/plain the results of the cost of their studies by choosing the number of study! courses they ta"e each term. Luestion 16&1C are based on the follo ing. Iardin argued that graBing land held in common $that is, open to any user% ould al ays be used less carefully than private graBing land. +ach rancher ould be tempted to overuse common land because the benefits ould accrue to the individual, hile the costs of reduced land #uality that results from overuse ould be spread among all users. 'ut a study comparing -17 million acres of common graBing land ith 0.. million acres of private graBing land sho ed that the common land as in better condition. 16. The ans er to hich of the follo ing #uestions ould be most useful in evaluating the significance, in relation to Iardins claim, of the study described above! $A% *id any of the ranchers hose land as studied use both common and private land! $'% *id the ranchers hose land as studied tend to prefer using common land over using private land for graBing! $)% 8as the private land that as studied of comparable #uality to the common land before either as used for graBing! $*% 8ere the users of the common land that as studied at least as prosperous as the users of the private land! $+% 8ere there any o ners of herds ho used only common land, and no private land, for graBing! $A% 8ith private graBing land, both the costs and the benefits of overuse fall to the individual user. $'% The cost in reduced land #uality that is attributable to any individual user is less easily measured ith common land than it is ith private land. $)% An individual ho overuses common graBing land might be able to achieve higher returns than other users can, ith the result that he or she ould obtain a competitive advantage. $*% 1f one user of common land overuses it even slightly, the other users are li"ely to do so even more, ith the conse#uence that the costs to each user out eigh the benefits. $+%There are more acres of graBing land held privately than there are held in common. -3. 1n tests for pironoma, a serious disease, a false
7C

GMAT

positive result indicates that people have pironoma hen, in fact, they do notE a false negative result indicates that people do not have pironoma hen, in fact, they do. To detect pironoma most accurately, physicians should use the laboratory test that has the lo est proportion of false positive results. 8hich of the follo ing, if true, gives the most support to the recommendation above!

CRITICAL REASONING TEST SECTION 30 MINUTES 20 QUESTIONS Luestions 1&- are based on the follo ing.

)ompanies , and 4 each have the same number of employees $A% The accepted treatment for pironoma does not ho or" the same number of have damaging side effects. hours per ee". According to $'% The laboratory test that has the lo est proportion records maintained by each of false positive results causes the same minor side company, the employees of effects as do the other laboratory tests used to )ompany , had fe er (ob& detect pironoma. related accidents last year than $)% 1n treating pironoma patients, it is essential to did the employees of )ompany begin treatment as early as possible, since even a 4. Therefore, employees of ee" of delay can result in loss of life. )ompany , are less li"ely to $*% The proportion of inconclusive test results is have (ob&related accidents than e#ual for all laboratory tests used to detect are employees of )ompany 4. pironoma. 1. 8hich of the follo ing, if $+% All laboratory tests to detect pironoma have the true, ould most strengthen same proportion of false negative results. the conclusion above! $A% )ompany 4 manufactures products that are more haBardous for or"ers to produce than does )ompany ,. $'% )ompany 4 holds more safety inspections than does )ompany ,. $)% )ompany 4 maintains a more modern infirmary than does )ompany ,. $*% )ompany , paid more for ne (ob&related medical claims than did )ompany
63

GMAT

.. 1n comparison to the standard type riter "eyboard, the +9), "eyboard, hich places the most&used "eys nearest the -. 8hich of the follo ing, if true, ould most ea"en the typists strongest fingers, conclusion above! allo s faster typing and results in less fatigue, Therefore, $A% The employees of )ompany 4 lost more time at replacement of standard or" due to (ob&related accidents than did the "eyboards ith the +9), employees of )ompany ,. "eyboard ill result in an $'% )ompany 4 considered more types of accidents to immediate reduction of typing be (ob&related than did )ompany ,. costs. $)% The employees of )ompany 4 ere sic" more often than ere the employees of )ompany ,. 8hich of the follo ing, if $*% Several employees of )ompany , each had more true, ould most ea"en the than one (ob&related accident. conclusion dra n above! $+% The ma(ority of (ob&related accidents at )ompany , involved a single machine. $A% 4eople ho use both standard and +9), "eyboards report greater difficulty in the transition from the +9), "eyboard to the standard "eyboard than in the transition from the standard "eyboard to the +9), "eyboard. $'% +9), "eyboards are no more e/pensive to manufacture than are standard "eyboards and re#uire less fre#uent repair than do standard "eyboards. $)% The number of businesses and government agencies that use +9), "eyboards is increasing each year. $*% The more training and e/perience an employee
61

4. $+% )ompany 4 provides more types of health&care benefits than does )ompany ,.

GMAT

has had ith the standard "eyboard, the more costly it is to train that employee to use the +9), "eyboard. $+% Novice typists can learn to use the +9), "eyboard in about the same amount of time it ta"es them to learn to use the standard "eyboard. Luestions 0&2 are based on the follo ing. Ialf of the sub(ects in an e/perimentthe e/perimental groupconsumed large #uantities of a popular artificial s eetener. After ard, this group sho ed lo er cognitive abilities than did the other half of the sub(ectsthe control group ho did not consume the s eetener. The detrimental effects ere attributed to an amino acid that is one of the s eeteners principal constituents. 0. 8hich of the follo ing, if true, ould best support the conclusion that some ingredient of the s eetener as responsible for the e/perimental results! $A% Most consumers of the s eetener do not consume as much of it as the e/perimental group members did. $'% The amino acid referred to in the conclusion is a component of all proteins, some of hich must be consumed for ade#uate nutrition. $)% The #uantity of the s eetener consumed by individuals in the e/perimental group is considered safe by federal food regulators. $*% The t o groups of sub(ects ere evenly matched ith regard to cognitive abilities prior to the e/periment. $+% A second e/periment in hich sub(ects consumed large #uantities of the s eetener lac"ed a control group of sub(ects ho ere not given the s eetener.

2. 8hich of the follo ing, if true, ould best help e/plain ho the s eetener might produce the observed effect! $A% The governments analysis of the artificial s eetener determined that it as sold in relatively pure form. $'% A high level of the amino acid in the blood inhibits the synthesis of a substance re#uired for normal brain functioning. $)% 'ecause the s eetener is used primarily as a food additive, adverse reactions to it are rarely noticed by consumers. $*% The amino acid that is a constituent of the s eetener is also sold separately as a dietary supplement. $+% Sub(ects in the e/periment did not "no hether they ere consuming the s eetener or a second, harmless substance. ;. Adult female rats ho have never before encountered rat pups ill start to sho maternal behaviors after being confined ith a pup for about seven days. This period can be considerably shortened by disabling the females sense of

6-

GMAT

smell or by removing the scent&producing glands of the pup. 8hich of the follo ing hypotheses best e/plains the contrast described above! $A% The sense of smell in adult female rats is more acute than that in rat pups. $'% The amount of scent produced by rat pups increases hen they are in the presence of a female rat that did not bear them. $)% 9emale rats that have given birth are more affected by olfactory cues than are female rats that have never given birth. $*% A female rat that has given birth sho s maternal behavior to ard rat pups that she did not bear more #uic"ly than does a female rat that has never given birth. $+% The development of a female ratNs maternal interest in a rat pup that she did not bear is inhibited by the odor of the pup.

7. The intervie is an essential part of a successful hiring program because, ith it, (ob applicants ho have personalities that are unsuited to the re#uirements of the (ob ill be eliminated from consideration. The argument above logically depends on hich of the follo ing assumptions! $A% A hiring program ill be successful if it includes intervie s. $'% The intervie is a more important part of a successful hiring program than is the development of a (ob description. $)% 1ntervie ers can accurately identify applicants hose personalities are unsuited to the re#uirements of the (ob. $*% The only purpose of an intervie is to evaluate hether (ob applicants personalities are suited to the re#uirements of the (ob. $+% the fit of (ob applicants personalities to the re#uirements of the (ob as once the most important factor in

6.

GMAT

ma"ing hiring decisions. C. :seful protein drugs, such as 6. An overly centraliBed economy, not the changes in the insulin, must still be climate, is responsible for the poor agricultural administered by the production in )ountry > since its ne government cumbersome procedure of came to po er. Neighboring )ountry J has in(ection under the s"in. 1f e/perienced the same climatic conditions, but hile proteins are ta"en orally, they agricultural production has been falling in )ountry >, it are digested and cannot reach has been rising in )ountry J. their target cells. )ertain nonprotein drugs, ho ever, 8hich of the follo ing, if true, ould most ea"en the contain chemical bonds that argument above! are not bro"en do n by the digestive system. They can, $A% 1ndustrial production also is declining in )ountry thus, be ta"en orally. >. $'% 8hereas )ountry J is landloc"ed, )ountry > has a The statements above most ma(or seaport. strongly support a claim that a $)% 'oth )ountry > and )ountry J have been research procedure that e/periencing drought conditions. successfully accomplishes $*% The crops that have al ays been gro n in )ountry hich of the follo ing ould > are different from those that have al ays been be beneficial to users of gro n in )ountry J. protein drugs! $+% )ountry >s ne government instituted a centraliBed economy ith the intention of ensuring $A% )oating insulin ith an e#uitable distribution of goods. compounds that are bro"en do n by target cells, but hose chemical bonds are resistant to digestion $'% )onverting into protein compounds, by procedures that or" in the laboratory, the nonprotein drugs that resist digestion $)% <emoving permanently from the digestive system any substances that digest proteins
60

GMAT

$*% *etermining, in a systematic ay, hat enBymes and bacteria are present in the normal digestive system and hether they tend to be bro"en do n ithin the body $+% *etermining the amount of time each nonprotein drug ta"es to reach its target cells. 13. )ountry J uses its scarce foreign&e/change reserves to buy scrap iron for recycling into steel. Although the steel thus produced earns more foreign e/change than it costs, that policy is foolish. )ountry Js o n territory has vast deposits of iron ore, hich can be mined ith minimal e/penditure of foreign e/change. 8hich of the follo ing, if true, provides the strongest support for )ountry Js policy of buying scrap iron abroad! $A% The price of scrap iron on international mar"ets rose significantly in 1C67. $'% )ountry Js foreign&e/change reserves dropped significantly in 1C67. $)% There is virtually no difference in #uality bet een steel produced from scrap iron and that produced from iron ore. $*% Scrap iron is no used in the production of roughly half the steel used in the orld today, and e/perts predict that scrap iron ill be used even more e/tensively in the future. $+% 9urnaces that process scrap iron can be built and operated in )ountry J ith substantially less foreign e/change than can furnaces that process iron ore.

11. @ast year the rate of inflation as 1.- percent, but for the current year it has been 0 percent. 8e can conclude that inflation is on an up ard trend and the rate ill be still higher ne/t year. 8hich of the follo ing, if true, most seriously ea"ens the conclusion above! $A% The inflation figures ere computed on the basis of a representative sample of economic data rather than all of the available data. $'% @ast year a dip in oil prices brought inflation temporarily belo its recent stable annual level of 0 percent. $)% 1ncreases in the pay of some or"ers are tied to the level of inflation, and at an inflation rate of 0 percent or above, these pay raises constitute a force causing further inflation. $*% The 1.- percent rate of inflation last year represented a ten&year lo . $+% Government intervention cannot affect

62

GMAT

the rate of inflation to any significant degree. 1.. 1n the :nited States in 1C6;, 1-. 'ecause no employee ants to be associated ith bad the average rate of violent ne s in the eyes of a superior, information about crime in states ith strict serious problems at lo er levels is progressively gun&control la s as ;02 softened and distorted as it goes up each step in the crimes per 133,333 persons about 23 percent higher management hierarchy. The chief e/ecutive is, therefore, less ell informed about problems at than the average rate in the lo er levels than are his or her subordinates at those eleven states here strict levels. gun&control la s have never been passed. Thus one ay The conclusion dra n above is based on the to reduce violent crime is to assumption that repeal strict gun control la s. $A% problems should be solved at the level in the management hierarchy at hich they occur 8hich of the follo ing, if $'% employees should be re arded for accurately true, ould most ea"en reporting problems to their superiors the argument above! $)% problem&solving ability is more important at higher levels than it is at lo er levels of the $A% The annual rate of management hierarchy violent crime in states $*% chief e/ecutives obtain information about ith strict gun&control problems at lo er levels from no source other than la s has decreased since their subordinates the passage of those la s. $+% some employees are more concerned about truth $'% 1n states ith strict gun& than about the ay they are perceived by their control la s, fe superiors individuals are prosecuted for violating such la s. $)% 1n states ithout strict gun&control la s, many individuals have had no formal training in the use of firearms. $*% The annual rate of nonviolent crime is lo er in states ith strict gun& control la s than in states ithout such la s. $+% @ess than half of the
6;

GMAT

individuals ho reside in states ithout strict gun& 12. The proposal to hire ten ne control la s o n a gun. police officers in 10. )orporate officers and directors commonly buy and Middleto n is #uite foolish. sell, for their o n portfolios, stoc" in their o n There is sufficient funding corporations. Generally, hen the ratio of such inside to pay the salaries of the sales to inside purchases falls belo - to 1 for a ne officers, but not the given stoc", a rise in stoc" prices is imminent. 1n salaries of additional court recent days, hile the price of M+GA )orporation and prison employees to stoc" has been falling, the corporations officers and process the increased directors have bought up to nine times as much of it caseload of arrests and as they have sold. convictions that ne officers usually generate. The facts above best support hich of the follo ing predictions! 8hich of the follo ing, if true, ill most seriously $A% The imbalance bet een inside purchases and ea"en the conclusion inside sales of M+GA stoc" ill gro even dra n above! further. $'% 1nside purchases of M+GA stoc" are about to $A% Studies have sho n that cease abruptly. an increase in a citys $)% The price of M+GA stoc" ill soon begin to go police force does not up. necessarily reduce crime. $*% The price of M+GA stoc" ill continue to drop, $'% 8hen one ma(or city but less rapidly. increased its police force $+% The ma(ority of M+GA stoc" ill soon be o ned by 1C percent last year, by M+GAs o n officers and directors. there ere 03 percent more arrests and 1. percent more convictions. $)% 1f funding for the ne police officers salaries is approved, support for other city services ill have to be reduced during the ne/t fiscal year. $*% 1n most :nited States cities, not all arrests result in convictions, and not all convictions result in
67

GMAT

prison terms. $+% Middleto ns ratio of police officers to citiBens has reached a level at hich an increase in the number of officers ill have a deterrent effect on crime. 1;. A recent report determined that although only three percent of drivers on Maryland high ays e#uipped their vehicles ith radar detectors, thirty&three percent of all vehicles tic"eted for e/ceeding the speed limit ere e#uipped ith them. )learly, drivers ho e#uip their vehicles ith radar detectors are more li"ely to e/ceed the speed limit regularly than are drivers ho do not. The conclusion dra n above depends on hich of the follo ing assumptions!

17. There is a great deal of geographical variation in the fre#uency of many surgical proceduresup to tenfold variation per hundred thousand bet een different areas in the numbers of hysterectomies, prostatectomies, and tonsillectomies. To support a conclusion that much of the variation is due to unnecessary surgical procedures, it ould be most important to establish hich of the follo ing!

$A% A local board of revie at $A% *rivers ho e#uip their vehicles ith radar each hospital e/amines the detectors are less li"ely to be tic"eted for records of every operation e/ceeding the speed limit than are drivers ho do to determine hether the not. surgical procedure as $'% *rivers ho are tic"eted for e/ceeding the speed necessary. limit are more li"ely to e/ceed the speed limit $'% The variation is unrelated to regularly than are drivers ho are not tic"eted. factors $other than the $)% The number of vehicles that ere tic"eted for surgical procedures e/ceeding the speed limit as greater than the themselves% that influence number of vehicles that ere e#uipped ith radar the incidence of diseases for detectors. hich surgery might be $*% Many of the vehicles that ere tic"eted for considered. e/ceeding the speed limit ere tic"eted more than $)% There are several categories of once in the time period covered by the report. surgical procedure $other than $+% *rivers on Maryland high ays e/ceeded the hysterectomies, speed limit more often than did drivers on other prostatectomies, and state high ays not covered in the report. tonsillectomies% that are often performed unnecessarily. $*% 9or certain surgical procedures, it is difficult to determine after the
66

GMAT

operation hether the procedures ere necessary or hether alternative treatment ould have succeeded. $+% 8ith respect to ho often they are performed unnecessarily, hysterectomies, prostatectomies, and tonsillectomies are representative of surgical procedures in general. 16. <esearchers have found that hen very over eight people, ho tend to have relatively lo metabolic rates, lose eight primarily through dieting, their metabolisms generally remain unchanged. They ill thus burn significantly fe er calories at the ne eight than do people hose eight is normally at that level. Such ne ly thin persons ill, therefore, ultimately regain eight until their body siBe again matches their metabolic rate. The conclusion of the argument above depends on hich of the follo ing assumptions! $A% <elatively fe very over eight people ho have dieted do n to a ne eight tend to continue to consume substantially fe er calories than do people hose normal eight is at that level. $'% The metabolisms of people ho are usually not over eight are much more able to vary than the metabolisms of people ho have been very over eight. $)% The amount of calories that a person usually burns in a day is determined more by the amount that is consumed that day than by the current eight of the individual. $*% <esearchers have not yet determined hether the metabolic rates of formerly very over eight individuals can be accelerated by means of chemical agents. $+% 'ecause of the constancy of their metabolic rates, people ho are at their usual eight normally have as much difficulty gaining eight as they do losing it.

1C. 1n 1C67 sinusitis as the most common chronic medical condition in the :nited States, follo ed by arthritis and high blood pressure, in that order. The incidence rates for both arthritis and high blood pressure increase ith age, but the incidence rate for sinusitis is the same for people of all ages. The average age of the :nited States population ill increase bet een 1C67 and -333. 8hich of the follo ing conclusions can be most properly dra n about chronic medical conditions in the :nited States from the information given above! $A% Sinusitis ill be more common than either arthritis or high blood pressure in -333. $'% Arthritis ill be the most common chronic medical condition in -333. $)% The average age of people suffering from sinusitis ill increase bet een 1C67 and -333. $*% 9e er people ill suffer

6C

GMAT

from sinusitis in -333 than suffered from it in 1C67. $+% A ma(ority of the population ill suffer from at least one of the medical conditions mentioned above by the year -333. -3. 4arasitic asps lay their eggs directly into the eggs of various host insects in e/actly the right numbers for any suitable siBe of host egg. 1f they laid too many eggs in a host egg, the developing asp larvae ould compete ith each other to the death for nutrients and space. 1f too fe eggs ere laid, portions of the host egg ould decay, "illing the asp larvae. 8hich of the follo ing conclusions can properly be dra n from the information above! $A% The siBe of the smallest host egg that a asp could theoretically parasitiBe can be determined from the asps egg&laying behavior. $'% Iost insects lac" any effective defenses against the form of predation practiced by parasitic asps. $)% 4arasitic asps learn from e/perience ho many eggs to lay into the eggs of different host species. $*% 9ailure to lay enough eggs ould lead to the death of the developing asp larvae more #uic"ly than ould laying too many eggs. $+% 4arasitic asps use visual clues to calculate the siBe of a host egg.

C3

Você também pode gostar